Domanda:
Credo di aver risolto un famoso problema aperto. Come convinco le persone sul campo che non sono un pazzo?
user3439590
2014-03-25 04:51:49 UTC
view on stackexchange narkive permalink

Mi interessa la situazione in cui hai un risultato molto interessante. Ad esempio, hai risolto un problema aperto molto importante. Tuttavia, non sei conosciuto nel campo e non hai pubblicazioni degne di nota. Il tuo supervisore pensa che il lavoro sia buono e tu sottoponi il lavoro a una rivista di alto profilo, ma vieni rifiutato.

Il fatto è che il contributo è molto forte. Rompe ciò che la maggior parte delle persone crede o ciò che ha già dimostrato: ad esempio, risolvi il problema P vs. NP o qualsiasi altro problema aperto ben noto.

I revisori respingono fortemente il tuo lavoro senza giustificazione e non dichiarano perché il risultato è sbagliato. Esempi di commenti del revisore includono:

  • "La prova deve essere sbagliata".
  • "Non puoi ottenere un risultato del genere."
  • "Non capisci bene la nozione di ..."

La mia domanda è cosa fare in questa situazione? Dove andare? Se il tuo consulente accetta il lavoro, ma i revisori della rivista principale rifiutano il lavoro senza nemmeno spiegare gli errori, cosa dovresti fare?

In tal caso, il supervisionato * probabilmente * ha torto nella percezione del proprio lavoro; rilevare questo è lo scopo della revisione tra pari. Ma poi ancora, forse no; forse la presentazione è semplicemente scadente o l'affermazione troppo oltraggiosa per alcuni cuori / menti. Carica su arXiv per il timestamp e continua a migliorare il modulo e l'invio. Il tuo nome non ha importanza (leggi: non dovrebbe) quando invii un articolo, quindi essere sconosciuto non è (leggi: non dovrebbe essere) un problema. Essere noto per roba a manovella semicotta, d'altra parte, è: evita di creare quell'impressione a tutti i costi!
Vedi anche [qui] (http://blog.computationalcomplexity.org/2009/01/so-you-think-you-settled-p-verus-np.html) e [qui] (http: //www.scottaaronson .com / blog /? p = 304) per ulteriori consigli. E ricorda che i premi Nobel e secoli di fama sono andati a persone che nessuno ha preso sul serio nel loro tempo (della vita).
Aggiungo che il "consiglio" che linko è ovviamente abbastanza supponente e va preso con le pinze. Per quanto ne sappiamo, * hai * la soluzione per un importante problema aperto. * Ma * devi tenere conto della situazione e presentare il tuo tentativo di conseguenza se vuoi che le persone lo prendano sul serio. I post del blog che collego dovrebbero darti un'idea di quanto possano essere permalosi gli esperti di dominio quando sono stati bombardati da (a loro) tentativi ovviamente sbagliati i cui autori non accettano "no" - per * decenni *. Scrivi per * loro *.
Dovresti leggere questa pagina: http: //research.microsoft.com/en-us/um/people/cohn/Thoughts/advice.html
1) Assicurati che le tue scoperte siano effettivamente corrette contattando altri pionieri nel campo (chiedi al tuo supervisore di farlo) 2) pubblicalo in Arxiv 3) aspetta gli inviti al seminario e il mondo della fama
@Raphael: Vi ricordo che il Premio Nobel è stato assegnato solo a persone viventi. Sottolineo anche che solo perché tutti hanno riso di Einstein non significa che se stanno ridendo di te, sei un nuovo Einstein. Buoni collegamenti però - grazie per quelli.
"Hanno riso di Columbus, hanno riso di Fulton, hanno riso dei fratelli Wright. Ma hanno anche riso di Bozo the Clown." -Carl Sagan (http://www.c2.com/cgi/wiki?TheyLaughedAtEinstein)
@EricLippert: [Nobel:] Ne sono consapevole; Stavo pensando ad alcuni artisti (e alla fama, non ai premi Nobel) quando ho messo la "(vita)" lì. [Inversione di argomento:] Ovviamente. Il fatto che i falsi negativi * accadano * è importante da tenere a mente, se non altro una quantità adeguata di cortese comune viene estesa a coloro che provano (e sembrano fallire).
@Raphael ["Detecting that is what peer review is for."] (Http://www.theguardian.com/technology/shortcuts/2014/feb/26/how-computer-generated-fake-papers-flooding-academia)
@JoelReyesNoche, Ovviamente, rispetto al signor Sagan, la gente non rideva di Colombo perché pensava che la Terra fosse rotonda, ma piuttosto perché sapevano che aveva drasticamente sottovalutato il diametro della Terra (risultando in un viaggio stimato di 3.700 km rispetto a quanto sarebbe stato un viaggio effettivo di 19.600 km). Colombo fu finanziato solo perché aveva promesso alla regina Isabella I e al re Ferdinando II di commerciare con l'Asia, in un momento in cui la coppia aveva un disperato bisogno del reddito dal commercio.
@Jigg "La prova deve essere sbagliata." "Non puoi ottenere tale risultato." "Non capisci bene il concetto di ..."
Se credi solo di averlo risolto, dovresti iniziare convincendoti prima di convincere gli altri. Solo quando hai scritto tutte le prove e i dettagli con una tale chiarezza che non ci sono assolutamente dubbi per te, e SAPI di aver dimostrato che dovresti iniziare a preoccuparti di convincere altre persone;) .. E tieni in mente che praticamente ogni il ricercatore ad un certo punto della sua carriera credeva di aver dimostrato alcuni Lemma / Proposizione / Teorema solo per rendersi conto in seguito di aver commesso un errore ...
a [crank] (http://en.wikipedia.org/wiki/Crank_ (person)) è definito da wikipedia come qualcuno che non è in grado di percepire di avere torto anche quando viene presentato con prove del contrario. non è necessario fare nulla se accetti la conclusione dei revisori. se non lo fai, stai chiedendo revisori che possano segnalare l'errore? e _puoi accettare che potresti sbagliarti e accettare prove contrarie? _ stackexchange ha forum di chat con alcuni esperti e suggerisce anche nel tuo caso [non menzioni l'informatica ma è nel tuo profilo] ti suggerisco di dare [cs.se] a tiro.
@Raphael Sbaglio che non puoi caricare nulla su arxiv se non hai un accreditamento universitario / centro di ricerca (se lavori ad es. Nel settore)?
Il problema principale qui è che hai fatto un'affermazione infondata, vale a dire: che non sei ** non ** un pazzo. Sì, ci sono alcuni casi (helicobacter, ad esempio) in cui ci sono voluti molto tempo e sforzi per cambiare un paradigma, ma quei casi sono famosi in parte per la loro rarità.
@ThorstenS .: Scusa, troppe negazioni per me (a quest'ora).
@PristineKavalostka: Sono a conoscenza di questo esempio, e sarei piuttosto imbarazzato se avessi partecipato al processo di revisione per alcuni anni. Detto questo, il fatto che * non * funzioni sempre bene (a causa della pigrizia, della pressione esterna e cosa no) non significa che * dovrebbe * non funzionare, ed è quello che ho affermato. (Probabilmente, (almeno) le pubblicazioni della conferenza sono del tutto prive di valore [in CS] e "tutti lo sanno". Ma, naturalmente, nessuno vuole ammetterlo apertamente per non parlare di agire, perché * tutti * coinvolti sono uno stakeholder.)
Un esempio di uno scienziato vincitore del Premio Nobel che è stato ridicolizzato è Shechtman per la sua scoperta dei quasicristalli. [1] Ha perso il suo lavoro di ricerca e gli è stato detto di leggere un libro di testo sui cristalli, simile a quello che ti è stato detto. Ci vuole perseveranza, convincere le persone 1 a 1 e trovare la rivista giusta in cui pubblicare. [1] http://www.bloomberg.com/news/2011-10-05/technion-s-shechtman-wins-chemistry-nobel -per-discovery-of-quasicrystals.html
@JoelReyesNoche: Hanno riso di Columbus ... e avevano ragione e Columbus aveva torto. (Sapevano tutti che la Terra è rotonda, Columbus ha semplicemente supposto che fosse 4 volte più piccola e quindi può attraversare l'oceano. È stato solo fortunato che ci fosse un po 'di terra in mezzo, altrimenti sarebbe morto di fame proprio come altri avevano previsto)
Forse mi manca qualcosa, ma se il problema è semplicemente non avere feedback, non potrebbe semplicemente pagare qualcuno per rivedere il suo lavoro? Certo che se lo paga, deve spiegargli cosa c'è che non va ...
vedi anche [analoghi di P vs NP nella storia della matematica] (http://mathoverflow.net/questions/160265/analogues-of-p-vs-np-in-the-history-of-mathematics) MO per qualche idea / analogie di come si relaziona a precedenti problemi molto difficili in matematica; ci sono altre questioni sollevate qui rispetto alla revisione tra pari. vedere anche la proposta area51 [stem-review] (http://area51.stackexchange.com/proposals/61370/stemreview); e anche questo saggio [Mostro matematico] (http://vzn1.wordpress.com/2012/10/18/math-monster/) contiene molti riferimenti sul problema P vs NP, incluso tra siti / domande di stackexchange e fornisce alcuni bkg su come / perché è _ così difficile_.
@Raphael - La prossima volta potresti prendere in considerazione la possibilità di pubblicare il tuo commento come risposta invece che come commento alla domanda.
Notato @eykanal:. Ho postato il commento su [cs.SE] e non ho dato per scontato che avesse un valore generale (nessuna restrizione a CS o P / NP qui). Ormai, ci sono risposte migliori, quindi lo lascio a quello (?).
Se la tua prova è effettivamente che "P = NP", allora il tuo percorso in avanti è semplice: scrivi un programma che risolva effettivamente i problemi NP in tempo polinomiale. Uno che hai dimostrato che puoi [rompere la crittografia a chiave pubblica] (http://security.stackexchange.com/a/12807), o calcolare alcuni [numeri Ramsey] sconosciuti (http://en.wikipedia.org/w/ index.php? title = Ramsey% 27s_theorem & action = edit§ion = 5), le persone ti crederanno.
@mbeckish Né questo funzionerebbe necessariamente (anche se l'algoritmo ha un runtime asintotico polinomiale, potrebbe non essere affatto pratico per input comuni) né questo convincerebbe necessariamente nessuno (un algoritmo può essere veloce in alcune istanze, ciò non significa che venga eseguito in tempo polinomiale).
@EricLippert Non strettamente vero; [se sono morti tra la nomina e il premio, possono ancora essere premiati dopo la morte] (http://en.wikipedia.org/wiki/Nobel_Prize#Posthumous_nominations)
@mbeckish: Raphael ha ragione; il tuo requisito è probabilmente sufficiente, ma non è necessario. Supponiamo che P = NP ma il risolutore SAT più veloce possibile venga eseguito in un tempo da n-kajillion; potrebbe ancora essere impossibile risolvere tutti i grandi problemi SAT in tempo polinomiale.
@Raphael - Sono sicuro che se funzionasse abbastanza bene con un numero sufficiente di input, incoraggerebbe le persone a esaminare la sua dimostrazione più attentamente.
@EricLippert - Vero. Basta buttare fuori un percorso sufficiente in avanti - sicuramente non l'unico percorso, e forse nemmeno un percorso fattibile. Non abbiamo assolutamente dettagli dall'OP.
Basta pubblicare il tuo documento sullo scambio di risma pertinente. Sarà abbattuto abbastanza presto.
@Rinze, [Shechtman] (http://en.wikipedia.org/wiki/Dan_Shechtman) aveva già letto quei libri di testo, in realtà aveva un dottorato di ricerca sull'argomento. Quindi il suo esempio non ha nulla a che fare con quando le persone dicono a coloro che non hanno letto un libro di testo di leggerne uno.
"Spezza ciò che la maggior parte delle persone crede o ciò che hanno già dimostrato" sembra abbastanza impressionante: quale problema hai risolto?
Risolvere P = NP spezzerebbe il mondo. Procederei in modo diverso risolvendo quel problema da altri grandi problemi. Per inciso, se non è ovvio per te, possiamo tranquillamente presumere che non capisci cosa sono P e NP.
Un altro link da aggiungere alle storie già discusse e su cui riflettere: http://www.sciencealert.com/a-purported-new-mathematics-proof-is-impenetrable-now-what
@djechlin: Dimostrare che P = NP non avrebbe alcun impatto pratico, perché provare che esiste un algoritmo che risolve un problema in tempo polinomiale non aiuta a trovare un algoritmo che risolva il problema nel corso della vita dell'universo.
Vorrei poter dare una taglia alla giustificazione di @E.P. per la sua taglia. Bello.
Forse dare qualche "suggerimento" su quale metodo hai usato e quale problema aperto hai risolto potrebbe aiutare la comunità qui a dire "beh, potrebbe essere vero" o "no, questo è un errore comune". Le domande sono così penzolanti nell'aria, è quasi impossibile non pensare che la tua affermazione sia sbagliata.
Abbiamo un sacco di esempi che i revisori non comprendono l'importanza del proprio lavoro.La CNN, che è una delle maggiori scoperte nel campo dell'apprendimento automatico e delle attività relative alla visione artificiale, è stata respinta nel CVPR.o anche il documento oscuro della conoscenza per Jeffry Hinton, uno dei padri fondatori del Deep Learning, era stato rifiutato.andate a vedere quanto sono stati (e lo sono tuttora) entrambi questi giornali.quindi sì, è qualcosa che ci si aspetta.
Interessante sapere come è finita questa ricerca rivoluzionaria?
Penso che la domanda sia qui "se tu [generico] accetti ciò che gli altri ti stanno dicendo che è sbagliato - _ dato che non hanno fornito alcun ragionamento effettivo e semplicemente mera affermazione_ - _on fede_, e rinunci a cercare di _comprendere_ quali sono effettivamente le obiezionie _perché_ è sbagliato? "O, in alternativa, "come puoi ottenere qualcosa di più concreto su _cosa_ c'è di sbagliato nel tuo articolo, partendo dal presupposto che ne leggano almeno una parte e che il loro rifiuto sia basato sull'individuazione di qualcosa di discutibile, ma che non sono riusciti a _comunicare_ effettivamente cosache _ è_ a te? "
Fai un passo indietro e chiediti se vivi in una terra di La-la.
Quattordici risposte:
Pete L. Clark
2014-03-25 07:25:32 UTC
view on stackexchange narkive permalink

La tua domanda presenta alcuni problemi. Date alcune domande che hai posto su altri siti SE negli ultimi giorni, ho qualche riserva sul fatto che la tua domanda venga posta in buona fede, ma presa per i suoi meriti è una domanda ragionevole, quindi cercherò di rispondere.

Il problema principale è che, anche ponendo questa domanda relativamente semplice, la tua scrittura è tutt'altro che chiara . Se non puoi scrivere chiaramente in questa situazione, le tue possibilità di scrivere un pezzo difficile di matematica o informatica teorica sono meno che buone. Ad esempio:

I suoi supervisori accettano il lavoro e lo hanno pubblicato in una rivista molto nota e vengono rifiutati.

Messa da parte problemi di accordo soggetto / verbo e coerenza del tempo, l'intera frase non ha senso: non puoi pubblicare un articolo e essere rifiutato.

Rompe ciò che la maggior parte delle persone crede

Non so cosa significhi "rompere ciò che la maggior parte delle persone crede".

o quello che hanno già dimostrato,

Cosa? Stai dicendo che la tua prova contraddice altri risultati comprovati? Presa alla lettera, ciò significherebbe che hai dimostrato che la matematica è incoerente. In pratica questo potrebbe significare solo che se il risultato è corretto, alcuni lavori pubblicati in precedenza non sono corretti. Se è così, allora devi essere molto chiaro e spiegare i difetti del lavoro precedente. Mi rattrista il fatto che tu non sembri crederci davvero, ma lo stai semplicemente buttando via come linguaggio sciolto.

cioè, risolve il problema P vs. NP o qualsiasi altro problema aperto ben noto.

Risolvere un problema aperto non "infrangerebbe ciò che le persone hanno già dimostrato" .... questo è ciò che significa che il problema sia aperto. Anche dire "P vs. problema NP o qualsiasi altro ben noto problema aperto" è un po 'strano di timidezza: non c'è nessun altro problema nell'informatica teorica (e pochissimi o nessuno nella matematica nel suo insieme) che è "come" P contro NP. Quindi non ha senso darlo come esempio. È come dire "cioè, ha trovato il Santo Graal o qualche altra coppa famosa".

In altre domande hai parlato specificamente di avere una prova di P contro NP e poi dopo aver interrogato ti sei ritirato da questo. Questo tipo di esitazione su ciò che hai fatto è una bandiera rossa di "irritabilità" che renderà i professionisti diffidenti.

I revisori rifiutano fermamente il proprio lavoro senza giustificazione e hanno detto che il risultato deve essere sbagliato.

Dire che il risultato deve essere sbagliato è non solo una giustificazione per il rifiuto, è la migliore giustificazione. Nessun revisore professionista dirà alla leggera che qualcosa non va. Quasi tutti i revisori che affermano questo indicheranno almeno un errore specifico. In caso contrario, in pratica quasi certamente significa che l'intero documento non aveva abbastanza senso per essere più specifico.

Se il tuo consulente accetta il lavoro, i revisori rifiutano il lavoro senza nemmeno spiegare gli errori (è il "miglior" diario nel suo dominio) allora cosa deve fare?

Se invii un articolo alla rivista più importante nel tuo campo sostenendo una soluzione al problema principale nel tuo campo e il tuo articolo non ha senso o non mostra nemmeno una corretta comprensione del problema, è probabile che gli editori non voler spendere molto tempo in risposta. D'altra parte, se sei sinceramente interessato a ottenere la loro esperienza, sembra ragionevole risponderti in modo molto educato e chiedere informazioni più specifiche sull'errore. Se la tua risposta è in qualche modo polemica, rischi che la redazione pensi che continuerai a perseguitarli all'infinito , e ad un certo punto devono smettere di rispondere. Quindi dovresti rispondere dicendo che non stai considerando di ripresentare il documento a quel diario, ma per i tuoi progressi sarebbe estremamente utile sapere cosa c'è di sbagliato in esso. Potresti anche menzionare che il tuo supervisore ha ritenuto che il documento fosse corretto.

In effetti potresti ricevere più aiuto su questo dal tuo supervisore. Se hai veramente "risolto il problema P vs. NP o qualsiasi altro problema aperto ben noto" e il tuo supervisore ritiene che la tua soluzione sia corretta, perché il tuo supervisore non sta spostando il cielo e la terra per essere sicuro che il tuo lavoro riceva l'attenzione che merita ? Questo non torna. Le due possibili spiegazioni sembrano essere (i) il tuo supervisore è stato troppo gentile con te: lui / lei non crede effettivamente di aver risolto P vs. NP; e (ii) l'imprimatur del tuo consulente non ha alcun peso nella comunità. Quest'ultimo purtroppo significa che la sua opinione sulla correttezza del tuo lavoro non vale molto.

Un buon modo per scoprire se è (i), (ii) o - ammetto che tutto è possibile! forse la rivista più importante nel tuo campo sta ignorando ingiustamente il tuo lavoro rivoluzionario - è cercare l'aiuto del tuo consulente per convincere un altro membro della facoltà a valutare il lavoro, preferibilmente qualcuno nel dipartimento con cui puoi parlare di recente.

Infine, sembra che tu abbia delle vere preoccupazioni che se una persona sconosciuta risolve un problema famoso, allora in qualche modo non conta. Questo non è davvero il modo in cui funziona il mondo accademico, a condizione che l'ignoto sia in grado di presentare il lavoro in un modo che abbia senso per gli esperti (e se no, che vergogna, ma che altro ci si può aspettare?). Hai sentito parlare del recente esempio di Yitang Zhang? Zhang era un docente non di ruolo presso l'Università del New Hampshire quando sbalordì il mondo matematico dimostrando l'esistenza di lacune primarie delimitate. Ha presentato il suo lavoro alla migliore rivista di matematica ... e a detta di tutti l'hanno accettato con una velocità insolita . In altre parole, hanno ricevuto un documento da qualcuno di cui probabilmente non avevano mai sentito parlare, l'hanno esaminato rapidamente e hanno visto che si trattava di un attacco plausibile a un enorme problema aperto, e di conseguenza sono entrati in azione molto più rapidamente e completamente che per la maggior parte dei contributi che ricevono. Questa è una storia incredibile, ma vera, e mostra come la community risponde a una situazione reale come questa.

Dato che io stesso non sono madrelingua inglese, tendo ad avere più tolleranza nei confronti della scrittura da parte di persone che non parlano inglese. Y. Zhang potrebbe non essere un buon esempio in questo caso. È andato negli Stati Uniti nel 1985 e ha conseguito il dottorato in matematica negli Stati Uniti. Da allora ha vissuto negli Stati Uniti. Da quello che so, Zhang non ha problemi in inglese mentre l'OP potrebbe avere seri problemi di lingua inglese. Tuttavia, sono d'accordo con molte parti della tua risposta.
@scaaahu: Non essere in grado di parlare bene l'inglese non è un difetto di carattere. È comunque un problema se l'inglese è la lingua in cui scrivi i tuoi articoli. C'è anche una distinzione da fare tra parlare una lingua in modo imperfetto ed esprimerti male. Nella mia risposta ho cercato di non insistere molto su questioni di grammatica e utilizzo.
Bene, in _I suoi / i suoi supervisori accettano il lavoro e lo hanno ** pubblicato ** in una rivista molto conosciuta_, la mia ipotesi è che la parola _pubblicato_ potrebbe essere l'uso improprio della parola inviata.
@scaaahu: Certo, l'OP ha usato del tutto la parola sbagliata qui. Ancora una volta, questo non è un crimine, ma farlo nel proprio lavoro accademico potrebbe certamente portare alla sua mancanza di comprensione e quindi al rifiuto. Guardala in questo modo: non sarebbe bello se le questioni linguistiche fossero la maggior parte delle cause delle difficoltà dell'OP? In tal caso, possono essere superati * a condizione che vengano riconosciuti e affrontati *. Non dire a qualcuno quando la loro scrittura non è chiara non significa fargli un favore, secondo me.
-1
@Raphael Non conosco l'ubicazione dell'OP quindi non posso rispondere a questa domanda per lui. Se fossi in lui, troverei editori professionisti per aiutarmi. Penso che la lingua sia solo uno dei suoi problemi. Potrebbero esserci anche altri problemi.
@Raphael Ho esaminato le domande dell'OP su altri siti SE. Quello su Signal Processing SE sembra a posto (non sono un esperto. Ho appena analizzato l'inglese). Le domande su TCS SE sembrano piuttosto terribilmente negative. Quello su Math SE è troppo breve ma ha centrato la chiave del problema. Sono confuso. Penso che possa scrivere in inglese in una certa misura. Il mio commento sopra voleva dire che l'inglese è probabilmente uno dei problemi che ha.
"Ha trovato il Sacro Graal o qualche altra famosa coppa" - frase fantastica.
@PeteL.Clark Sebbene tutte le sue domande si riferiscano a P vs NP, penso che potrebbe effettivamente avere qualche risultato per il limite di Shannon o il canale di interferenza. Vedi le sue domande nel gruppo di elaborazione del segnale http://dsp.stackexchange.com/questions/15206/can-we-break-the-shannon-capacity
Congratulazioni, hai (involontariamente, ne sono sicuro) convinto l'OP che se aggiusta il suo inglese tutto andrà bene :) (vedi il suo commento sulla mia risposta)
@Selfishness: "[I] Se una persona non è conosciuta e ha fatto una cosa molto buona, ignorerà il suo lavoro (almeno all'inizio)." Dato che la mia risposta conteneva un controesempio specifico, recente e chiaro a questo, non sono sicuro di cosa aggiungere in risposta.
@ff524: Sì, non era quello il mio intento, ma penso che vada bene: indipendentemente dal fatto che i problemi linguistici siano o meno la totalità o il peso del problema, sarà comunque vantaggioso per l'OP affrontarli. (In effetti, solo così sarà in grado di capire se è effettivamente così.)
sospetto che questa risposta sia così apprezzata in parte perché l'intervistato è così altamente qualificato per scriverla, lavorando / affermandosi nel mondo accademico "nel modo più duro". ecco un po 'di bkg e un'ampia raccolta di ref su [Zhang twin prime breakthru] (http://vzn1.wordpress.com/2013/10/04/zhang-twin-prime-breakthru-vs-academic-trackgrind/) & refs su cosa vuol dire lavorare nel mondo accademico e le sfide lì & intervista con Zhang ecc
Almeno, l'ho valutato molto bene sul contenuto, non avendo riconosciuto l'autore e non preoccupandomi di inseguire il puntatore ora.
Tutte le domande collegate nella tua seconda frase sono state rimosse, per la cronaca.
Potrebbe valere la pena notare che la maggior parte delle riviste che ho presentato finora accetta solo documenti resi anonimi. Quindi, l'argomento se le persone sconosciute alla comunità siano trattate ingiustamente non ha alcun peso in un argomento a favore o contro il rifiuto.
@EricTobias questo varia ampiamente a seconda del campo. Nell'informatica teorica, le revisioni in cieco sono la norma.
Non ero sicuro se fare +1 per le risate al "Sacro Graal o qualche altra coppa famosa", o -1 per l'ira che dovrò sicuramente affrontare dopo aver risvegliato l'intero pavimento proprio ora con risate di risate .. Alla fine Ho deciso -1 + 1 = 0, ma +1 per il consiglio del paziente.
Se la prima lingua della persona non è l'inglese, perché non può pubblicare prima su un giornale nella sua lingua madre? Ci sono pubblicazioni accademiche non inglesi e non statunitensi
@DVK: Se le tue abilità in inglese non sono abbastanza forti e non hai esperienza di scrittura in inglese, scrivere prima il tuo articolo nella tua lingua madre è probabilmente una buona idea. Tuttavia, quando si tratta di pubblicazioni, il numero di lingue in cui le rispettabili riviste di matematica / TCS accettano documenti è ormai piuttosto ridotto: puoi presentare un articolo scritto in inglese o francese praticamente ovunque; alcune prestigiose riviste internazionali accettano documenti tedeschi [sebbene la maggior parte dei madrelingua tedeschi ora scriva tutti i loro articoli in inglese].
Ci sono paesi con comunità matematiche straordinariamente grandi e forti - Cina, Giappone, Russia - in cui praticamente tutti i ricercatori seri pubblicano i loro articoli in inglese o francese. * Ci * sono * alcune riviste che pubblicano articoli in altre lingue, ma questi articoli di solito non vengono letti dalla comunità più ampia. Se vuoi solo pubblicare qualcosa, ok; ma se vuoi essere riconosciuto per aver risolto un problema famoso, questo tipo di pubblicazione è problematico: vedi ad es. http://www.newscientist.com/article/dn24915-kazakh-mathematician-may-have-solved-1-million-puzzle.html#.U-423aPbH-4.
Inoltre, dici in tutte le capitali che ci sono pubblicazioni accademiche non statunitensi: con chi stai parlando? Più della metà di tutte le mie pubblicazioni sono su riviste che hanno sede in paesi diversi dagli Stati Uniti o che non sono fondamentalmente nemmeno in un determinato paese.
@PeteL.Clark - ci sono paesi al di fuori degli USA. La Russia ha pubblicazioni di matematica (almeno le tornava quando volevo controllare). Sono sicuro che lo sia anche la Cina. Non ho detto che deve essere una pubblicazione internazionale.
@DVK: Se non "ti interessa controllare" qual è la situazione attuale nelle pubblicazioni di matematica in lingue diverse dall'inglese, perché la stai commentando? Ho spiegato perché per risolvere un famoso problema è importante pubblicare in una lingua che gli esperti del settore sappiano leggere. Non convinci le persone che non sei un pazzo pubblicando un giornale che nessuno degli esperti leggerà. Se il tuo punto è semplicemente che "Ci sono paesi al di fuori degli USA": te lo prometto, lo so.
@PeteL.Clark - stai insinuando che la sua unica opzione è pubblicare in un inglese perfetto. A meno che la lingua madre non sia di qualche rara tribù in Amazzonia, NON è così. Può pubblicare nel suo paese natale e, se è davvero un documento valido, farlo tradurre professionalmente. Ci sono esperti nel campo che leggono in lingue diverse dall'inglese, ve lo assicuro.
@DVK: Non sto insinuando ciò che stai suggerendo. Inoltre, quello che ho dato è il mio sincero consiglio su come essere preso sul serio al meglio per risolvere un problema difficile. Ti ho fornito un collegamento specifico a qualcuno il cui lavoro non viene preso sul serio per questo motivo esatto. Dato che i tuoi commenti recenti non sono in sintonia con le sfumature delle mie risposte, smetterò di risponderti su questo punto. Ma se il tuo consiglio è diverso, non esitare a lasciare una risposta. Il miglior consiglio è supportato da prove esterne ed esperienza personale, quindi sarei molto interessato a saperne di più.
I collegamenti alle domande ora sono morti.
Parlando di inglese e chiarezza del testo, la domanda di OP mi è arrivata come una domanda ipotetica "E se qualcuno ..." invece di "Affermo di avere ...", questa risposta sembra presumere che OP abbia effettivamente a che fare con ilquestione ha rappresentato, piuttosto che chiedere come vanno le cose in questa situazione (per altre persone).
@Mefitico, Non so se la tua reputazione è sufficiente per vedere la versione originale del post, ma nella prima versione, l'OP afferma "P.S. Questa è una $ \ textbf {real} $ situazione."
"Nessun revisore professionista dirà alla leggera che qualcosa non va".Non è vero nella mia esperienza.Ho ricevuto più volte affermazioni che una prova è sbagliata quando chiaramente non lo è (e successivamente l'ho pubblicata altrove).Anche prove molto brevi.Sembra che accada virilmente anche nelle riviste di alto livello.Forse perché sono i più inondati di cattive proposte che indovinare qualcosa non va in base al tuo istinto, mentre un comportamento pigro e inaccettabile, è probabilmente una buona ipotesi.
"In pratica questo potrebbe solo significare che se il risultato è corretto, alcuni lavori pubblicati in precedenza non sono corretti."Crede fermamente nella coerenza di (alcuni sistemi di base sufficientemente espressivi di) matematica pura?Altrimenti, non mi è chiaro perché potrebbe significare solo quello che dici potrebbe significare.
ff524
2014-03-25 06:19:35 UTC
view on stackexchange narkive permalink

Indipendentemente dal fatto che il lavoro sia corretto o meno, si applica la seguente dichiarazione:

L'onere della prova spetta all'autore convincere il lettore del risultato.

La comunità (ad esempio, editori, revisori) non ha la responsabilità di valutare il tuo lavoro in modo soddisfacente. Se i revisori hanno fatto uno sforzo in buona fede per leggere il tuo articolo e non erano convinti, allora devi rendere la tua argomentazione più convincente.

(Questo non significa, apportare alcune modifiche banali e reinviare. Ciò significa, dimostra i tuoi risultati in modo così completo e con dettagli così atroci e con una comprensione così dimostrabile del contesto del problema, che diventano indiscutibili. Quindi trova un modo per esprimere i risultati in modo convincente.)

Se mentre lo fai trovi un errore, beh, saresti in buona compagnia.

+1. "Affermazioni straordinarie richiedono prove straordinarie." Se non è convincente, fallo e ricorda che tutto ciò che serve è un controesempio per dimostrare che non hai risolto il problema, quindi devi davvero considerare ogni possibile caso limite prima di poter avanzare tale affermazione. Se c'è un'eccezione, non l'hai risolta ma potresti aver risolto un sottoinsieme ... che può essere o meno una nuova informazione.
@user3439590 Se i revisori comprendono il contributo ma pensano che l'inglese dovrebbe essere migliorato, scriveranno qualcosa come "Questo documento fornisce un contributo utile, ma presenta problemi di scrittura in inglese". Se i revisori non lo hanno scritto, non li hai convinti di aver dato un contributo utile.
@Selfishness_has_equilibrium tieni presente però che è anche possibile che la barriera inglese ti porti a non capire alcune cose sottili con alcune definizioni e nozioni. Il che potrebbe ulteriormente portare facilmente a risolvere una versione leggermente diversa del problema ... E specialmente nella teoria dei grafi, a volte un piccolo dettaglio è la differenza tra un problema aperto e un facile esercizio in un corso introduttivo ...
@Selfishness_has_equilibrium, se le scarse competenze in inglese ostacolano la capacità dei revisori di capire quello che stai cercando di dire, ovviamente lo rifiuteranno. Ciò vale il doppio per le affermazioni straordinarie (e il triplo se rivendichi la persecuzione o ti paragoni a Einstein). Ti suggerirei di collaborare con qualcuno _ che ha eccellenti competenze in inglese_ per rivedere / controllare il tuo lavoro e modificarlo per leggibilità. Il costo potrebbe essere che (a seconda di quanto contribuiscono) dovrai elencarli come co-autori.
"Se nel farlo trovi un errore, beh, saresti in buona compagnia." Mi ha ricordato [questa notizia] (http://www.youtube.com/watch?v=nQGvXx4rfUY) (Youtube)
Il problema è che, poiché non sai cosa c'è che non va o perché la tua prova non è convincente, allora sei effettivamente lasciato a indovinare a caso e mi sembra che l'ipotesi casuale incontri contraddizioni con altri principi come "non sprecaretempo".Anche se suppongo che se non vogliono dirtelo, allora sì, indovina a caso e "sprecare" il loro tempo con 3-4 proposte più completamente riviste prima di arrendersi, perché devi risolvere la contraddizione in qualche modo e questo significa che tudevi bruciare un principio xo l'altro e sembra meglio bruciare il minore che il maggiore.
@keshlam: '"I reclami straordinari richiedono una prova straordinaria."' - concordato. "Se non è convincente, fallo" - Concordò.Il problema è che devi prima saperlo, perché non è convincente ed è impossibile leggere la mente di qualcuno che letteralmente non ti dà nient'altro con cui lavorare se non "È sbagliato".La domanda di OP, credo, riguarda _interamente_ quel componente della _ mancanza di conoscenza_.Come si migliora - o addirittura si rifiuta _ ragionevolmente_ la propria follia - il proprio lavoro quando non si ha proprio niente con cui continuare a dirti _ cosa_ c'è di sbagliato?
Chiedi a qualcuno di insegnarti.Questo inizia ammettendo che potresti essere completamente sbagliato.Se non puoi permetterti di prendere lezioni, controlla e / o ottieni consigli per i libri di testo correnti e assumi tutoraggio quando rimani bloccato.Se non ne vale la pena, perché qualcuno dovrebbe ascoltare?
Kaveh
2014-03-27 03:17:08 UTC
view on stackexchange narkive permalink

In primo luogo, assicurati di non essere davvero un pazzo prima di provare a convincere gli altri. Leggi queste caratteristiche comuni delle pedivelle. Se si applicano a te, chiedi aiuto professionale.

Per il resto della risposta, presumo che tu abbia davvero risolto un famoso problema aperto. Di seguito "lui" si riferisce a una tipica affermazione non esperta per avere una soluzione per un famoso problema aperto e "lei" si riferisce a un esperto dell'argomento.

  1. Non esiste una scorciatoia facile per te! Se stai cercando una scorciatoia semplice e facile per far verificare la tua soluzione da un esperto, allora questa risposta non fa per te e posso assicurarti che quello che vuoi non accadrà.

  2. Comprendi l'entità del tuo reclamo!
    Ad es Se stai affermando di avere una prova che P non è uguale a NP, allora sei il ragazzo che afferma di avere un progetto per un razzo che può essere costruito con la tecnologia e le risorse attuali per portare un essere umano su Andromeda e tornare in sicurezza mentre gli esperti stanno avendo difficoltà a mandare un essere umano su Marte. Se stai affermando di avere una prova che P è uguale a NP, allora sei la persona che afferma di avere una macchina per i viaggi nel tempo.

  3. Capisci perché gli esperti sono riluttante a coinvolgere direttamente non esperti.
    Molti esperti sarebbero interessati a conoscere qualsiasi progresso importante nel loro campo. Per esempio. ci sono teorici della complessità che leggono ogni documento relativo a P vs. NP pubblicato su arXiv (arXiv ha una politica di accettazione molto indulgente per quanto riguarda le affermazioni di P vs. NP). fai sapere agli altri esperti se notano qualcosa di interessante. Ma

    • Non sei l'unico con tali affermazioni.
      Ci sono migliaia di persone che fanno regolarmente tali affermazioni.

    • Tutti i precedenti soffrivano di problemi banali che nessun esperto avrebbe creato.
      Il tuo lavoro è dimostrare che non sei uno di loro.

    • Il suo tempo è prezioso.
      Per la maggior parte non è realmente monetario. Ma penso che dare dei numeri sarebbe utile. Nella mia università uno studente laureato viene pagato più di $ 40 l'ora per contrassegnare semplici incarichi universitari, questo non è niente in confronto a quanto un esperto potrebbe addebitare per una consulenza nel settore.

    • I non esperti spesso mancano delle competenze e delle conoscenze di base per comprendere le sue risposte.
      Ad es gli manca la maturità matematica, non conosce le definizioni e la terminologia di base, ecc. Non è raro che un esperto dica a un non esperto ciò che ha non è una prova. Lei non significa che la dimostrazione non è corretta, significa che non è nemmeno una prova, nel senso che una mela non è una prova. Non capisce quando gli viene detto che è " nemmeno sbagliato! ". Per fargli capire la sua risposta, avrebbe dovuto insegnargli quelle abilità e conoscenze richieste, troppo lavoro solo per convincerlo che non ha una soluzione. Spesso non è paziente né interessato all'apprendimento (ad esempio leggere un libro di testo), è interessato solo a una conferma di ciò che crede essere una soluzione. Troppo lavoro in quel caso.

    • Spesso è impossibile soddisfarlo.
      A causa dei punti sopra menzionati, spesso insiste sulla validità della sua affermazione anche dopo che lei gli dice che non lo è. Altre volte, quando capisce la risposta, la considera un semplice errore di facile correzione, non fondamentale. Cerca di risolverlo e di convincerla a verificarlo. Questo porta avanti e indietro.

    • Sottovaluta il tempo e lo sforzo necessari da parte sua per rispondere alla sua richiesta.
      Pensa che per lei sia un lavoro semplice e facile rispondere alla sua richiesta. Per esempio. si aspetta che lei gli dia un controesempio in cui il suo algoritmo fallisce. Trovare un controesempio per un algoritmo è un compito molto difficile (come saprebbe chiunque abbia contrassegnato algoritmi universitari o incarichi di teoria della complessità). Trovare una spiegazione del motivo per cui un'idea è fondamentalmente difettosa e non può funzionare è ancora più difficile.

  4. Non capisce che non è un enigma.
    Non è interessata alla domanda solo per se stessa. La soluzione al problema sarà accompagnata da importanti progressi nel suo campo I teorici della complessità non si preoccupano di P contro NP solo per se stessi; si aspettano che la soluzione per P contro NP arriverà con importanti progressi nella nostra comprensione della natura del calcolo efficiente e dei suoi limiti. Spesso lui non lo capisce. considera la domanda come un gioco o un puzzle che pensa di aver vinto e basta. Questo atteggiamento è frustrante per gli esperti.


Ora ecco alcuni suggerimenti :

  1. Sii umile.
    È molto più facile convincerla a dare un'occhiata alla tua soluzione se sei sinceramente umile e desideroso di imparare e accettare se ti viene detto che hai torto.

  2. Assicurati di aver compreso cosa è necessario per risolvere la domanda.
    Ad es. capire che un programma che sembra risolvere in modo efficiente un problema NP-completo non è una prova, capire che un'idea non fa una prova, assicurarsi di aver compreso le definizioni e la terminologia, ecc.

  3. Conoscere le basi.
    Continuo a ripetere: leggi un buon libro di testo sull'argomento e risolvi i suoi esercizi. È vantaggioso per te perché ne saprai di più e sarai più convincente. È utile per lei perché non sprecherai il suo tempo con semplici errori che avresti notato tu stesso se avessi letto un buon libro di testo. È fastidioso trattare con persone che affermano di aver risolto P vs. NP ma ripetutamente commettono errori di base che un bravo studente che ha seguito un corso universitario sull'argomento non farà.

  4. Usa il tuo vero nome.
    Il mancato utilizzo del tuo vero nome indica che stai cercando di evitare di subire qualsiasi potenziale conseguenza negativa del fatto che la tua affermazione non sia corretta. Usare il tuo vero nome indica che sei abbastanza sicuro da essere pronto a subire potenziali conseguenze professionali negative se ti sbagli, così puoi essere preso più sul serio. Se non sei completamente sicuro della tua richiesta, non farle perdere tempo.

  5. Non eludere il lavoro. Fai la tua parte prima di aspettarti l'aiuto degli altri.
    Se vuoi che lei esamini la tua soluzione, dovresti dedicare 10 volte più tempo e sforzi di quanto lei spenderà per aiutarti. Per reclami su P contro NP devi fare molto di più.

  6. Non avrai più di una possibilità.
    Fallo valere . Se sulla prima pagina del tuo articolo trova uno stupido errore o un errore di base (ad esempio, non conosci nemmeno le definizioni di P e NP), le tue affermazioni saranno finite per sempre.

  7. Comprendi gli ostacoli noti per risolvere la domanda e perché non si applicano alla tua soluzione.
    Ad es. se stai affermando che P non è uguale a NP, dovresti avere una buona idea del perché le barriere di rilaltivazione e prove naturali non si applicano alla tua soluzione. Allo stesso modo, se stai affermando che P è uguale a NP.

  8. Prova a dimostrare affermazioni più semplici e accettabili.
    Ad es. se hai una prova di P è uguale a NP, dovresti anche avere una prova di risultati maggiori più semplici e deboli come Factoring è in P. Se puoi estrarre una prova pulita per tali affermazioni, puoi prima provare a pubblicarle. essere molto più facile da verificare in quanto sono considerati più probabili.

  9. Assicurati che la tua soluzione non sia troppo efficace.
    In altre parole, assicurarsi che non sia in contraddizione con altri risultati noti. Per esempio. se il tuo argomento per P è uguale a NP mostrerebbe anche che P è uguale a ExpTime (che sappiamo essere falso) allora sei nei guai (Scott Aaronson menziona alcuni altri casi di risultati troppo forti nel suo post sul blog Otto Signs A Claimed P ≠ NP Proof Is Wrong).

  10. Verifica la tua soluzione.
    Assicurati che non ci siano errori. Tutti i passaggi dovrebbero essere facilmente visti come seguire da quelli precedenti. Assicurati di non fare supposizioni extra in nessun momento.

  11. Ricontrolla la tua soluzione.
    Metti da parte completamente la tua prova per due o più settimane . Non pensarci. Quindi torna indietro e ricontrolla con una mente fresca come se stessi controllando la soluzione di qualcun altro.

  12. Crea prove per le tue affermazioni.
    Ad es. se hai un algoritmo veramente efficiente (cioè il suo tempo di esecuzione è un polinomio con piccole costanti) che hai dimostrato di risolvere un problema NP-completo, allora non dovrebbe essere un compito difficile battere lo stato- of-art risolutori SAT o per rompere vari protocolli crittografici basati su congetture di durezza (queste congetture saranno false se P è uguale a NP).

  13. Scrivi un riassunto e un'introduzione chiari e concisi di facile lettura.
    Non inserire alcun background / storia / conseguenze filosofiche / discussioni importanti / commenti generali non necessari. È un famoso problema aperto; ogni esperto conosce il suo significato. Salvali per la tua versione finale. In questo momento dovresti concentrarti sul convincerla che la tua affermazione è corretta. Lei vuole prima un breve di facile lettura, senza errori e convincente. spiegazione di livello della tua soluzione. Dovrebbe anche spiegare perché gli eventuali ostacoli noti non si applicano alla tua soluzione e dovrebbe contenere anche qualsiasi altra prova che possa supportare la correttezza della tua affermazione. Se fallisci, è improbabile che il lettore continui a leggere.

  14. Assicurati che il resto del tuo articolo corrisponda al tuo abstract e introduzione.
    fallisci, è improbabile che il lettore continui a leggere.

  15. Assicurati che ogni dettaglio del tuo articolo sia corretto.
    Segui la struttura standard di Documenti sull'argomento. Controlla alcuni famosi documenti ben scritti nell'area che hanno risolto i principali problemi aperti. Tutte le definizioni dovrebbero essere chiare, facili da capire e rigorose. Ogni teorema (lemma, ecc.) Dovrebbe essere chiaramente e rigorosamente affermato, e la dimostrazione di ciascuno di essi dovrebbe seguire la loro affermazione. Dovrebbe essere in grado di vedere perché ogni affermazione nella dimostrazione è corretta in base ai passaggi, alle definizioni e ai lemmi precedenti senza troppi problemi. Se fallisci, è improbabile che il lettore continui a leggere.

  16. Chiedi a un esperto generale che personalmente sappia che controlli la tua soluzione.
    Presumo che tu non conosca personalmente nessun esperto nell'area della questione. Più vicino è l'esperto generale all'area della domanda, meglio sarà. Per esempio. per P vs. NP, puoi chiedere a un matematico, preferibilmente un informatico teorico. Il parere di persone che non sono esperti nell'argomento potrebbe non avere molto peso, ma ti assicurerà che non stai commettendo qualche semplice errore.
    Capire che a questo punto qualcuno che non ti conosce personalmente non ha motivo di verificare la tua soluzione.

  17. Avere un altro esperto generale che ti conosca personalmente controlla la tua soluzione.
    Roma non è stata costruita in un giorno. Devi acquisire fiducia nella tua soluzione a poco a poco. Coloro che convinci possono diventare i tuoi ponti per raggiungere gli esperti.

  18. Se sono convinti chiedi loro di mostrare il tuo soluzione a un esperto che conoscono.
    Ad es per P contro NP, chiedi loro di mostrarlo a un teorico della complessità che conoscono. A questo punto è meno probabile che tu commetta un errore di base e hai buone prove a sostegno della tua affermazione. La tua soluzione ora richiede l'esperienza di un esperto dell'argomento.

  19. Se è convinta, la mostrerà sicuramente ad altri esperti.
    Le notizie su qualsiasi progresso importante in un'area si diffonderanno molto rapidamente tra gli esperti in quell'area. Altri esperti (teorici della complessità nel caso P vs. NP) ricontrolleranno la tua soluzione in modo indipendente. Se sono convinti, probabilmente riceverai un invito a inviare il tuo articolo a una famosa rivista (qualcosa come JACM nel caso di P vs. NP).

  20. Non pretendere di risolvere un famoso problema aperto più di una volta.
    Come ho scritto sopra, non avrai più di una possibilità! Non hai il diritto di chiederle di vedere cosa c'è di sbagliato nella tua soluzione fissa se hai commesso un errore (l'eccezione è quando lei esplicitamente ti chiede di provare a correggere la tua soluzione e inviare a lei.)

  21. Non aspettarti una spiegazione del motivo per cui la tua idea non può funzionare.
    È improbabile che qualcuno possa per dimostrare formalmente che un'idea informale non può funzionare. Se l'idea è sufficientemente formale, la ragione per cui non può funzionare può essere di per sé un nuovo risultato interessante; tuttavia, provare tali risultati può essere ancora più difficile che risolvere la domanda originale. Nel caso di P vs NP, se stai affermando di avere un algoritmo efficiente per un problema NP-hard non dovresti aspettarti che trovi un input dove il tuo algoritmo fallisce.


In sintesi,

Capisci che non è tenuta ad aiutarti. Se ti sta aiutando, lo sta facendo per generosità. Ha il diritto di fermarlo ogni volta che le piace senza alcuna spiegazione Sii consapevole del suo tempo, non sprecarlo per quello che avresti potuto / dovuto fare da solo, cerca di rendere il suo lavoro nell'aiutarti il ​​più semplice possibile e non fare nulla che la farà pentire di aver provato per aiutarti.

Cosa intendi con "capire che non è un puzzle"?
@Jack, ecco cosa intendevo: alcune persone trattano P vs. NP come un gioco per computer di un livello con una risposta Sì o No che devono vincere. Noi (teorici della complessità) ci preoccupiamo di P vs NP perché crediamo che la risoluzione della questione sarà accompagnata da progressi significativi nella nostra comprensione della natura del calcolo efficiente e dei suoi limiti, non ci interessa solo per il suo bene. Come scrisse una volta Scott: "_ [Ci] piace prendere P vs. NP come il nostro" esempio di punta "di una vasta classe di domande su ciò che è e non è fattibile per i computer, a nessuna delle quali sappiamo come rispondere ._ "
"se hai un algoritmo che hai dimostrato di risolvere un problema NP-completo, non dovrebbe essere un compito difficile da battere ..." - questo potrebbe essere un cattivo consiglio. Non c'è motivo di pensare che un tale algoritmo debba essere "reale" efficiente: potrebbe avere prestazioni pessime su tutti gli input che possiamo memorizzare. Altrimenti, buona risposta. Dimostra perfettamente che ci vuole uno sforzo straordinario per risolvere un problema straordinario. Ovviamente, uno stravagante a) non sarebbe in grado di diagnosticare se stesso eb) confuterebbe molti dei tuoi punti perché hanno queste teorie del complotto. (* Tu * presumi che tutti siano amichevoli.)
+1: che risposta eccezionalmente dettagliata e utile. Spero che l'OP apprezzi la generosità del tempo e dello spirito che sono stati coinvolti in questo.
@Raphael, hai ragione in linea di principio (anche se in pratica accade raramente), ho cercato di rendere l'affermazione più precisa.
@Kaveh: Penso che sia concepibile che un testimone di P = NP (se esiste) * è * esattamente un tale algoritmo. Spiegherebbe sicuramente perché nessuno ne ha trovato uno finora. Grazie per aver apportato il cambiamento.
Un altro punto sulla lista di controllo potrebbe essere che dovresti parlare del problema usando qualunque notazione sia più standard, dare una citazione molto standard a un'affermazione del problema aperto in cui viene usata quella notazione e astenersi dall'introdurre cumuli della tua notazione per le cose finché non diventano chiaramente necessarie.
In realtà non è vero che un algoritmo asintoticamente migliore per un problema difficile renda facile battere le implementazioni software esistenti di altri algoritmi. Ad esempio, c'è questo algoritmo in tempo lineare per la triangolazione poligonale, asintoticamente migliore del metodo semplice O (n log (n)) time - ma buona fortuna implementarlo in un ragionevole lasso di tempo! Per non parlare del fatto che le implementazioni esistenti spesso comportano molte modifiche e scorciatoie utili per casi comuni / piccoli, ecc. Quindi, ritrarrei questo suggerimento a meno che OP non sia un programmatore esperto.
Questa è una magnifica risposta! Tutti coloro che lavorano al problema dovrebbero leggere questo. È come l'altro, più famoso, "come dire se sembri un pazzo" su Internet, solo molto più costruttivo e utile. Spero che questa risposta trovi un vasto pubblico nel gruppo target, anche se non saprei quale sia il modo migliore per ottenerlo. La promessa 'Presumo che tu abbia davvero risolto un famoso problema aperto' a cui sei all'altezza, si spera assicurerà che le persone prendano davvero a cuore il tuo consiglio (specialmente perché ovviamente nella maggior parte dei casi non l'hanno fatto ma non sono ancora affrontalo).
Il tuo punto (2) è magnifico.
Questa risposta è eccezionale ed è diventata la mia prima risorsa su questo argomento. Avere una taglia! (che assegnerò al termine della settimana di pubblicità gratuita)
+1 per i punti secondari in 3. Tuttavia, non credo che il consiglio iniziale sulla lettura delle caratteristiche comuni delle pedine per l'autodiagnosi sia efficace. Tutte le pedivelle che conosco sostengono che non si applica a loro (ad esempio, Dunning-Kruger).
Tuttavia, se qualcuno ha già _detto_, dopo aver ricevuto il documento, che è sbagliato, ciò suggerisce di aver visto almeno un errore _actual_ e _concrete_ [probabilmente significativo] in esso.Quindi non c'è bisogno di chiedere loro una "valutazione specializzata" o di "pagarli per sborsare ore di tempo".Se l'hanno capito, lo hanno comunque rivisto e hanno detto che era sbagliato, _qualcosa_ li ha informati, e quello che OP vuole sapere, penso, è _ cosa fai quando quel qualcosa non è stato comunicato_.L'unico "lavoro" aggiuntivo necessario da parte del revisore sarebbe quello di _comunicare_ di più, non di analizzare ulteriormente l'articolo.
A volte qualcosa è così incoerente che non è possibile indicare nessuna parte che è sbagliata.È ciò che chiamiamo "non è nemmeno sbagliato", "non analizza nemmeno".A volte, sulla base di precedenti esperienze negative con tali autori, l'esperto teme di segnalare un problema particolare in quella situazione perché spesso tali autori lo trattano come se fosse un problema da risolvere e riprovare, mentre ciò che lei sta dicendo non è "questo è_il_ problema con la tua soluzione "ma" qui è solo uno dei problemi con la tua soluzione, e ce ne sono molti altri simili ".
Per problemi di matematica, è possibile avere una prova formale e convalidarla utilizzando un correttore di bozze come Coq, un autore può spendere tempo e formalizzare e convalidare la dimostrazione senza un esperto.Fare appello a un'esportazione è una scorciatoia per ottenere l'accettazione di una prova meno formale.Un esperto deve giudicare la correttezza in un breve periodo di tempo.Alcuni problemi comuni sono che l'autore fraintende le definizioni o presenta lacune significative nei loro argomenti.In quello successivo, ad es.può solo dire "A non segue da B", il che significa che c'è un divario significativo nella dimostrazione (diverso dal dire "A-> B è falso").
Come ha detto @Kimball, quasi tutte le pedivelle (escluse quelle veramente incrinate che balbettano solo senza senso) soffrono di Dunning-Kruger e non ammetteranno mai di essere una manovella.Mi ricorda un'estensione incoerente T di PA che insiste sul fatto che essa stessa è coerente, non importa quanto rigorosamente dimostri dalle stesse ipotesi di T che T è sbagliato.
Suresh
2014-03-25 05:28:22 UTC
view on stackexchange narkive permalink

Se la tua interpretazione degli eventi è: "Ho un lavoro straziante di genio sbalorditivo e l'unico ostacolo all'accettazione è che non sono molto conosciuto e le élite stanno bloccando il mio lavoro", allora è improbabile che tu riceva buoni consigli su cosa fare qui o altrove.

Il problema, come indica Raphael, è che mentre è possibile che questa interpretazione sia corretta, è molto più probabile che in realtà il tuo risultato NON risolva il principale problema aperto che tu penso di sì.

Una volta ammesso che esiste questa possibilità, si presentano molti passaggi, tutti elencati negli ottimi collegamenti forniti. Raggiungere le persone che potrebbero commentare il tuo lavoro, guardare la letteratura per vedere se approcci come il tuo sono stati provati prima e hanno fallito, vedere se la tua soluzione risolve anche problemi correlati (più semplici) e così via.

sì. pochi lo menzioneranno e forse è considerato un tabù farlo, ma come accennato nella tua risposta sembra in alcuni che queste affermazioni grandiose potrebbero essere correlate in modo sostanziale con sintomi / problemi psicologici, ad esempio [delusioni di grandezza] (http: //en.wikipedia .org / wiki / Grandiose_delusions), [narcisism] (http://en.wikipedia.org/wiki/Narcissism), [disturbo narcisistico di personalità] (http://en.wikipedia.org/wiki/Narcissistic_personality_disorder) ecc
Questa può essere anche una caratteristica del disturbo bipolare. (Questo non vuol dire che un ricercatore con disturbo bipolare non possa ottenere risultati eccellenti. È solo per indicare una bandiera rossa che suggerisce un possibile problema.)
Nate Eldredge
2014-03-25 07:19:45 UTC
view on stackexchange narkive permalink

Cosa dice il tuo consulente di tutto questo?

Se crede davvero che tu abbia risolto questo grave problema, dovrebbe spostare le montagne per aiutarti a pubblicarlo e diffonderlo. (Sembra che ci sia anche il suo nome, quindi ha un incentivo ancora maggiore.) Ma hai usato la frase piuttosto tiepida che lei "accetta". È meglio che prima sia completamente d'accordo o che spieghi più dettagliatamente le sue riserve (che potrebbero rivelarsi in realtà difetti fatali).

Il vantaggio che hai rispetto alla media è che, come studente, hai già legami con la comunità scientifica, tramite il tuo consulente. Approfitta di questo. Una volta che tu e il tuo consulente siete sicuri che il vostro manoscritto è della migliore qualità possibile (vedere la risposta di ff254), pubblicatelo su arXiv e diffondete. Il tuo consulente sicuramente conosce gli esperti del settore e dovrebbe avere una reputazione sufficiente per farli interessare.

Non sono sicuro del tuo campo, ma in matematica, questo attualmente tende ad essere il modo in cui la comunità gestisce le soluzioni ai principali problemi aperti. Non lo invii solo agli Annals , chiedi agli arbitri di approvarlo e poi attendi qualche mese finché tutti ricevono il loro numero di Annali per posta e rimangono sbalorditi. Invece, fai in modo che la comunità lo studi prima . Convinci alcuni esperti che è abbastanza plausibile da meritare la loro attenzione e lo guardano. O trovano subito un difetto critico (il caso più comune), oppure trovano un difetto minore che tu o qualcun altro risolvi e forse, gradualmente, si sviluppa un consenso che probabilmente è giusto. Questo è quando lo invii a Annals.

Una cosa che mi preoccupa in quello che hai scritto è:

Rompe ciò che la maggior parte delle persone crede o ciò che ha già dimostrato ...

Qual è? La distinzione è fondamentale. Se contraddice l'intuizione delle persone, questo alza un po 'l'asticella, ma gli scienziati sono abituati a sorprendersi. Se contraddice qualcosa precedentemente dimostrato, ciò alza molto il livello. Ti mette l'onere non solo di mostrare che il tuo lavoro è giusto, ma di mostrare specificamente perché il lavoro precedentemente accettato era effettivamente sbagliato. (Non puoi semplicemente dire "Il mio è giusto, quindi il loro deve essere sbagliato.") Non hai detto nulla sull'aver fatto ciò. (E se non puoi trovare un difetto nel lavoro precedente, allora la tua affermazione è in realtà sulla falsariga di "La matematica è incoerente". è più o meno sulla luna.)

Ho anche lasciato una risposta simile, anche fino alla frase (scelta in modo indipendente) "spostare le montagne" rispetto a "spostare il cielo e la terra". Devo dire però che convincere la comunità a studiare il tuo lavoro rivoluzionario è un buon modo di andare e comune, ma non credo che avvenga sempre in questo modo, soprattutto tra le persone che hanno pochi legami con esperti qualificati. Nella mia risposta ho menzionato l'esempio (straordinario) di Yitang Zhang. Per quanto ne so, ha davvero lavorato in isolamento e ha inviato il suo articolo agli Annals piuttosto che guardarlo spesso.
@PeteL Clark: Heh. Grandi menti, ecc. Grazie per la tua bella risposta, l'esempio di Zhang, e soprattutto per la decostruzione della domanda.
@NateEldredge "La barra su [dimostrare che la matematica non è coerente] è più o meno sulla luna" Nota che la matematica * non * può essere dimostrata coerente, quindi in realtà la convinzione che la matematica sia coerente è anche solo intuizione :) Naturalmente, la comunità non è abituato a essere * così * sorpreso!
La tua risposta mi ricorda una [storia davvero interessante] (https://golem.ph.utexas.edu/category/2011/09/the_inconsistency_of_arithmeti.html). Versione breve: un matematico serio pensa di aver trovato una prova che la matematica (PA) è incoerente, la spiega abbastanza chiaramente che gli esperti possono capire, un matematico di alto livello legge lo schema e individua l'errore, e l'autore ritira la dimostrazione. Quindi anche affermazioni davvero straordinarie verranno considerate se vengono spiegate in modo chiaro e ragionevole da una persona ragionevole.
@IstvanChung bene, tecnicamente, non è coerente _ per definizione_?
@Lohoris Vedi [il secondo teorema di incompletezza di Gödel] (http://en.wikipedia.org/wiki/Godel%27s_incompleteness_theorem#Second_incompleteness_theorem).
Potrebbe essere possibile dimostrare che la matematica è coerente, ma solo se è incoerente
@NoahSnyder: Commento in ritardo, ma ho cercato di rintracciare quel caso / collegamento per mesi / anni. Grazie a Dio l'hai incluso qui!
rumtscho
2014-03-28 01:52:34 UTC
view on stackexchange narkive permalink

Attaccando la tua stessa prova ancora più forte degli altri.

Seriamente, c'è un motivo per cui le persone nella tua disciplina non sono state in grado di trovare la risposta per secoli. La probabilità a priori che tu abbia torto è così alta che anche quando hai creato una bella prova, la probabilità a posteriori che tu abbia ragione è troppo bassa. Ciò significa che, se conosci abbastanza della tua disciplina, non dovresti essere convinto di averla risolta. Dato un problema che ha resistito a lungo alla soluzione, essere convinti di averlo risolto solo perché hai una prova in cui credi è un segno sicuro di un fesso.

Quindi, in questo caso, un comportamento non pazzo sarà quello di provare a smontare la prova, abbatterla, farla a pezzi da tutte le angolazioni possibili. Questo è ciò che faranno i tuoi colleghi, ed è ciò che si aspettano che tu faccia. Dimenticare il tuo orgoglio, i tuoi pregiudizi soggettivi ed essere spietato nei confronti del tuo risultato.

Ti credono solo dopo che hai trovato più modi per confutare il tuo risultato di quanto loro stessi possano pensare, li hai provati tutti e hai fallito in tutti. E il tuo articolo deve mostrare chiaramente che questo è quello che hai fatto. Qualsiasi altra cosa ti farà guadagnare il titolo di crackpot.

Semplicemente geniale. Non l'avrei formulato meglio io stesso ... l'idea mi è venuta in mente quando ero sul mio primo "punto di svolta scientifica"; Mi sono fatto una doccia fredda, un serio rimprovero, una forte critica al valore della soluzione - e solo allora mi sono reso conto che sebbene la mia soluzione sia * irrilevante per il problema in questione e non lo risolva *, presenta un potenziale per risolvere * altri problemi correlati *. Per ottenere ciò, bisogna sapere come fallire ...
mirkastath
2014-03-25 11:24:58 UTC
view on stackexchange narkive permalink

Vorrei aggiungere alla risposta esauriente di Nate Eldredge che, se il tuo lavoro scuote o frantuma le opinioni comunemente condivise nella tua comunità, allora è molto importante che tu riconcili queste opinioni con le tue, con questo intendo: mostra esattamente dove si trova la comunità è "sbagliato" o "non esattamente giusto" e perché. Offri controesempi, previsioni, tutto ciò che puoi.

La relatività non sarebbe da nessuna parte se non si riducesse alla buona vecchia meccanica newtoniana dove quest'ultima funzionava perfettamente!

E a volte è appropriato spiegare perché * sembrava * che ciò che è stato fatto non potesse essere fatto.
usul
2014-03-25 07:23:21 UTC
view on stackexchange narkive permalink

Un consiglio è di controllare molto attentamente che le prove siano corrette, chiedere consiglio al proprio supervisore e cercare terze opinioni. Forse il supervisore ha colleghi nell'area di ricerca che sarebbero disposti a leggere la bozza e offrire un feedback concreto.

Se la rivista inviata è buona, i revisori non hanno fornito alcun feedback utile, allora c'è quasi certamente un problema con l'abstract e l'introduzione.

L'abstract e l'introduzione dovrebbero rendere chiara la nuova idea che consente questo "passo avanti". Presumibilmente molti hanno affrontato questo problema in passato e hanno fallito; ci possono essere convinzioni diffuse sul motivo per cui è difficile dimostrare o forse anche conosciute "barriere" per tentare prove. L'abstract e l'introduzione dovrebbero menzionare chiaramente e brevemente perché tali convinzioni, obiezioni o barriere non si applicano o come sono state superate.

In breve, l'abstract e l'introduzione devono dare al lettore scettico motivo di credere al documento potrebbe essere corretto, date le conoscenze di base del lettore. In tal caso, mi auguro che i revisori menzionino almeno il motivo per cui non credono al risultato.

Thorsten S.
2014-03-27 22:40:09 UTC
view on stackexchange narkive permalink

Anche se la maggior parte delle risposte sembra avere molta fiducia nel sistema accademico, vorrei offrire un altro punto di vista.

Penso che in realtà sia molto più difficile per uno sconosciuto (per un campo specifico) per rappresentare una soluzione alla comunità scientifica di quanto normalmente previsto.

Gli scienziati fanno casini ea volte in modo regale.
Primo esempio: il famigerato problema di Monty Hall.
Oltre il 65% di tutte le risposte professionali a Marilyn (con tutte tipi di accademici, inclusi gli statistici) hanno respinto con forza le loro risposte, a volte senza giuste scherni e provocazioni, inclusi Paul Erdos e Straight Dope Cecil Adams, quindi anche la maggior parte degli esperti può fallire.

Secondo esempio: il famigerato neutrino anomalia La cosa interessante qui non è l'errore in sé, ma la reazione suArxiv. Chiunque avesse avuto il coraggio di offrire teorie superluminali prima che l'annuncio sarebbe stato immediatamente dichiarato come un fanatico della relatività. Dopo l'annuncio, i documenti si sono riversati nell'offrire ogni sorta di teorie superluminali che spiegavano ciò che ora sappiamo essere semplicemente un cavo.

Quali sono i problemi che uno sconosciuto può incontrare?

  1. Arxiv. Hai bisogno di un'affiliazione da un'università o di un istituto di ricerca e / o l'approvazione di un autore conosciuto. Arxiv può revocare o limitare il tuo accesso senza spiegazioni. Questo requisito si applica anche a scienziati pienamente qualificati che lavorano in aziende.

  2. Riviste. Troppe persone cercano di pubblicare i propri risultati su un numero troppo esiguo di riviste rispettabili. Anche le riviste sono piuttosto ridotte, devi aspettare molto tempo per essere pubblicate. Le riviste meno conosciute possono avere barriere più basse, ma hai il reale pericolo che il contributo venga perso. E anche i giornalieri inferiori potrebbero rifiutare il documento.

  3. Scienziati. La situazione è diversa nei vari paesi, ma normalmente gli scienziati sono oberati di lavoro e sottopagati. Non hanno il tempo né le risorse per esaminare i contributi con la minima possibilità di ottenere un jackpot scientifico.

Se qualcuno pensa che il punto di vista non sia valido, prova solo per divertimento a fornire un normale documento sotto uno pseudonimo e l'indirizzo di casa.

L'unica opzione praticabile che vedo è contattare gli scienziati sul campo e cercare di lavorare con loro sul contributo che potrebbe essere più difficile di quanto sembri. L'elenco fornito da Kaveh è una buona risorsa con cui iniziare.

La tua valutazione del problema di Monty Hall non è corretta. Il problema di fondo era che il problema non era ben specificato. La sottile ambiguità nella specifica era ciò che ha causato il disaccordo tra gli esperti.
@EnergyNumbers Sì, ho sentito questa scusa. Ma la domanda originale non offre ambiguità. L'host conosce la posizione dell'auto (non è casuale), è uno spettacolo di giochi (non può aprire la portiera della macchina) e sceglie * un'altra * porta con un * vuoto *. E, in realtà, quello che quasi tutti i critici hanno detto è stato * non *: "Ha due diverse soluzioni, la risposta dipende dalle seguenti circostanze, ecc. Pp.". Quello che hanno detto per lo più è stato: "LA POSSIBILITÀ È SEMPRE 1/2, STUPIDO STUPIDO !!" Sia Massimo Piattelli-Palmarini che Gero von Randow, un giornalista scientifico, hanno spiegato la soluzione e le persone ancora non erano d'accordo.
Per fermare solo una discussione in corso: La mia tesi è semplicemente che il consenso scientifico * può * rovinare regalmente. Se non credi che ciò possa accadere, cerca alcune delle citazioni originali che eminenti scienziati fecero sulla deriva dei continenti di Wegener e sulla materia oscura di Fritz Zwicky al momento della scoperta. Credimi, non vuoi leggere questo.
Il risultato del neutrino è stato un caso molto speciale. È stato annunciato con molta, molta cautela, quasi come "Ehm, ragazzi? Abbiamo questo risultato un po 'pazzo che non possiamo spiegare. Abbiamo provato per anni e abbiamo trovato errori e risolti, ottenendo comunque la stessa cosa. Potete vedere ragazzi. cosa abbiamo sbagliato? " Con quella quantità di attenzione prestata nel lavoro sperimentale che sembrava forse mostrare neutrini più veloci della luce, diventa degno di considerazione. La maggior parte dei risultati FTL sono solo qualcuno che dice: "Sì, ma se Einstein avesse torto e ..." o, peggio, "Einstein dice che è impossibile ma, se lo ridefiniamo sottilmente, allora ..."
@DavidRicherby E poiché hanno chiesto cosa hanno fatto di sbagliato, le persone dovrebbero aspettare e inviare proposte invece di presentare documenti FTL. Violare la relatività è un problema * davvero terribile *. Tutte le particelle hanno massa a riposo "normale" (m = 0 => v = c / m> 0 => v
Philip Gibbs
2014-03-25 16:28:41 UTC
view on stackexchange narkive permalink

Nonostante quello che dirà la gente, è vero che le riviste rifiuteranno gli articoli che utilizzano il profilo dell'autore senza una revisione adeguata. È difficile dire quanti documenti vengono rifiutati in questo modo, ma Elsevier afferma di rifiutare dal 30 al 50 percento dei documenti senza una revisione per altri "motivi tecnici". Vedi anche questo documento su come gli editori possono risparmiare tempo esaminando attributi dell'autore come l'affiliazione per rifiutare i documenti senza guardarli.

Ho esperienza personale di questo perché di recente fatto progressi significativi su un noto problema aperto vecchio di 100 anni dopo che esperti del settore avevano affermato che i progressi futuri sarebbero stati probabilmente molto lenti. La rivista a cui ho inviato il documento l'ha rifiutata non appena ho confermato di non essere affiliata. Non c'era alcun rapporto del revisore e non hanno fornito alcun motivo specifico. Avevo rispettato tutti i loro requisiti tecnici per la presentazione.

Tuttavia, ho fatto presente loro che secondo il codice di condotta del comitato etico della pubblicazione a cui la rivista afferma di aderire "Le decisioni editoriali non dovrebbero essere influenzate dalle origini del manoscritto" e "Le riviste dovrebbero avere un meccanismo dichiarato per gli autori di appellarsi contro le decisioni editoriali". Con mia sorpresa hanno risposto dopo un po 'di tempo per dirmi che lo avrebbero esaminato di nuovo.

È vero che ci sono molte risoluzioni dichiarate di problemi come P vs NP che possono essere ignorate a colpo d'occhio . Questo può essere fatto perché ci sono ragioni ben note per cui questi problemi sono difficili e una soluzione dovrebbe essere risolta. Molte prove dichiarate di problemi aperti da parte di non accademici discendono rapidamente in un linguaggio non standard che rende difficile persino affrontare il motivo per cui hanno torto, quindi vengono semplicemente ignorati dalla comunità. Spetta agli autori assicurarsi di comunicare correttamente le proprie idee.

Se hai una soluzione a un problema aperto, il mio consiglio è di inviarlo a un repository aperto come arXiv. Se non riesci a ottenere un endorser usa viXra o figshare (divulgazione completa: sono un amministratore di viXra) Non prestare attenzione alle cose negative dette su viXra. Il suo scopo è solo quello di darti un timestamp indipendente e una copia archiviata a cui puoi puntare. Non cerca di rivedere o dare credibilità al tuo lavoro in alcun modo. L'ultima cosa che dovresti fare è sottoporla a riviste o inviarla a esperti senza averne una copia pubblica verificata perché se si tratta davvero di una svolta c'è un rischio reale di plagio che può essere scongiurato solo archiviando una copia precedente.

Il tuo primo paragrafo sembra indicare che il collegamento è a un editore di Elsevier che ammette di utilizzare la profilazione dell'autore per rifiutare i documenti, ma non c'è nulla del genere in quel collegamento (elenca ragioni tecniche perfettamente valide per rifiutare un articolo). Nota anche che nella comunità matematica, mettere qualcosa su viXra ti segnerà come meno serio (che sia così o meno), quindi non consiglierei a nessuno di caricare qualcosa lì a meno che non abbia davvero altre opzioni.
Il mio primo paragrafo non dice quello che pensavi. Stai leggendo cose tra le righe che non ci sono. Se rivendichi una prova di un problema noto come autore non affiliato, avrai già problemi a essere preso sul serio e troverai molto difficile caricare su arXiv. Anche se hai un endorser, arXiv manterrà sempre un contributo non affiliato per giorni in attesa della revisione del moderatore e potrebbe quindi rifiutarlo esponendo l'autore a un possibile plagio. viXra caricherà molto rapidamente senza tale moderazione. Anche figshare sembra funzionare velocemente se preferisci.
Sono pienamente consapevole di ciò che dice la prima riga. Ti sto menzionando quello che potrebbe essere visto come dire a qualcuno che lo legge per la prima volta.
Lo modificherò cercando di renderlo più chiaro
fyi sulla base del suo profilo PG elenca il suo sito web come [Vixra Blog] (http://blog.vixra.com) sembra essere affiliato al sito [Vixra] (http://blog.vixra.org/), un'alternativa ad arXiv, che è nascosto nella risposta
Devi leggere di nuovo la risposta dove dice "divulgazione completa: sono un amministratore di viXra" Nota anche che non è un sito commerciale. Non abbiamo bisogno di reclamare per affari. Lo cito insieme a un'alternativa solo perché è una parte rilevante della risposta.
* "se si tratta davvero di una svolta c'è un rischio concreto di plagio" *. Questo è il genere di cose che spesso fa identificare le persone come pazzi: credono che i redattori e i revisori ruberanno il loro lavoro e allo stesso tempo lo rifiuteranno come non valido. Credi davvero che questo sia un rischio quando ti sottoponi a riviste rispettabili e sei a conoscenza di casi in cui è successo?
Sì, credo che ci sia un rischio e anche se è piccolo, è qualcosa che gli autori dovrebbero adottare per evitare. Non sono a conoscenza di casi identificabili, ma ciò non toglie la possibilità e il rischio che ciò accada. Il plagio nel mondo accademico sta diventando così palese che ora puoi trovare documenti che apparentemente sono stati copiati quasi interamente parola per parola tre volte vedi http://academia.stackexchange.com/questions/18512/how-can-i- trovare-l'articolo-originale-di-un-lavoro-spesso-plagiato Sarebbe ingenuo pensare che questa non sia la punta dell'iceberg.
Peggio: gli articoli sono * accettati * a causa del profilo dell'autore. Detto questo, se devi selezionare 20 articoli su un numero sempre crescente di contributi (le persone in alcune aree sono arrivate a pubblicare come i conigli procreano) posso capire perché sceglieresti 50 documenti che presumi siano buoni da * alcuni * criterio e farli esaminare solo. Capisci, non mi piace.
Sì, è comprensibile se gli editori vogliono risparmiare tempo, ma sarebbe disonesto farlo e poi affermare di aderire al Codice di condotta COPE.
@PhilipGibbs Il post che colleghi non riguarda il plagio da parte di revisori o editori di riviste rispettabili che hanno rifiutato il materiale e poi lo hanno spacciato per proprio. Quindi, ancora una volta, sei a conoscenza di casi in cui ciò è accaduto?
David, il rischio che accada esiste indipendentemente dal fatto che sia mai accaduto o che sia stato documentato che sia accaduto. Il consiglio che do a chiunque faccia una scoperta è di non correre il rischio se apprezza la priorità del proprio lavoro.
@PhilipGibbs Certamente. Allo stesso modo, esiste anche il rischio che gli alieni spaziali possano rubare il tuo lavoro e usarlo per produrre un raggio della morte che ci uccide tutti, indipendentemente dal fatto che ciò non sia mai accaduto. La domanda è se questi rischi siano abbastanza grandi da essere preoccupati. L'entità dei rischi * non * è indipendente dal fatto che si siano mai verificati o meno.
Ho trovato un paio di file di casi in cui i revisori sono stati accusati di plagio del lavoro che avrebbero dovuto rivedere. La conclusione in entrambi i casi è stata che si trattava solo di una mancata dichiarazione di conflitto di interessi quando stavano lavorando a ricerche simili. In un caso (http://publicationethics.org/case/parallels-between-unpublished-manuscript-and-published-article-other-authors) il documento è stato rifiutato dal revisore e nell'altro (http: // publishingethics. org / caso / revisore-autore-conflitto-interesse) semplicemente non aveva fatto la revisione.
David, se pensi che i rischi non siano alti va bene per te. Non vorrei correre il rischio.
@jwg Questo rischio esiste e non aiuta a ridicolizzarlo. Vedere [Jocelyn Bell] (https://en.wikipedia.org/wiki/Jocelyn_Bell) e [Rosalind Franklin] (https://en.wikipedia.org/wiki/Rosalind_Franklin). Come studente, il nostro gruppo ha scoperto che un assistente ha utilizzato i dati di un altro assistente senza autorizzazione. Mark Chu-Carroll di "Good Math, Bad Math" ha avuto un brutto incontro quando ha parlato con un'altra persona di una nuova idea e ha scoperto in seguito che questa persona ha pubblicato un documento con questa idea senza riconoscerla.
@ThorstenS., Come ha sottolineato David Richerby, elencare vari casi di plagio non equivale a citare un singolo caso in cui revisori o editori hanno rifiutato il materiale presentato solo per plagiarlo. Non ho "ridicolizzato" questo - ho sottolineato, spero utilmente, che questo tipo di affermazione è esattamente ciò che le persone cercano per riconoscere le pedine. Se stai presentando un lavoro serio, ti stai rendendo un disservizio apparendo irritabile. D'altra parte, se il tuo lavoro presentato è stato veramente plagiato, dovresti fare storie al riguardo.
@jwg Bene, eccoci qui: gli autori americani Vijay R. Soman e Philip Felig dell'Università di Yale hanno fatto * esattamente * questo: rifiutando un articolo di un gruppo NIH e rubando i loro dati per i loro scopi. I professori tedeschi di cancro Herrmann e Brach hanno respinto un desiderio di sovvenzione da un gruppo olandese, hanno tradotto gli argomenti originali dall'olandese al tedesco e hanno chiesto con questo per la propria borsa di studio.
@ThorstenS. In realtà trovo abbastanza confortante che il miglior esempio sembri essere quello di più di 30 anni fa, e che non è nemmeno il plagio del revisore (piuttosto, non è riuscito a segnalare un possibile conflitto di interessi e ha mostrato l'articolo in esame a un subordinare che poi plagiato esso).
@TobiasKildetoft Erm, Herrmann / Brach è del 2003. In anticipo: penso che la maggior parte degli scienziati sia onesta. Ma a parte il fatto che il palo è ora spostato (10/100/1000 casi?), Penso di dover menzionare che le università potrebbero essere citate in giudizio per rimborsare completamente le sovvenzioni se viene rilevata una frode. Un altro fatto interessante potrebbe essere che le frodi (come Jan-Hendrik Schön) sono per lo più scoperte da soffiate anonime, il che potrebbe avere qualche motivo perché Reiner Protsch ha distrutto le carriere di diverse persone che hanno cercato di mettere in guardia altri scienziati.
@ThorstenS. Giusto, ma quello non era un revisore di un nuovo risultato, quella era una situazione leggermente diversa.
Ho avuto a che fare sia con le pedine che con i veri casi di plagio - esistono entrambi.Il fatto che, se sei fortunato, non avrai un incidente stradale non significa che dovresti essere sbadato.Posso simpatizzare con qualcuno che crede di avere un risultato rivoluzionario che vuole proteggere la sua priorità (vedi anche Perelman vs Chern dove anche essere visto come competente e il giornale pertinente essere disponibile abbastanza presto non ha impedito una disputa sulla priorità).I ricercatori non affiliati o debolmente affiliati sono particolarmente vulnerabili.Ho i miei dubbi sugli eccentrici, ma non ignoriamo il pericolo di plagio.
@CaptainEmacs Mi dispiace, sembra che non ci siano Perelman vs Chern (che è morto nel 2004), e il motivo dell'isolamento di Perelman sembra essere ingiustificato (non credo totalmente ai rapporti sul New Yorker).
Michael Wehar
2015-02-05 11:51:26 UTC
view on stackexchange narkive permalink

Dalla mia esperienza di studente laureato, sembra che i ricercatori appassionati del loro lavoro facciano più o meno la stessa cosa indipendentemente dalla sua presunta importanza .

Ecco cosa fare:

(1) Scrivi il tuo lavoro nel miglior modo possibile discutendo le idee e i concetti con i tuoi amici, collaboratori, colleghi, ecc.

(2) Promuovi il tuo lavoro ad altre ricerche, amici, collaboratori, ecc. Ma spesso le persone sono impegnate, quindi mantieni basse le tue aspettative e alta la cortesia.

(3) Individua il tuo pubblico e cerca il riviste, workshop e conferenze appropriati a cui sottoporsi.

(4) Se hai domande o dubbi prima di inviare, contatta qualcuno affiliato alla conferenza. Ho avuto un'esperienza positiva in questo modo, ma a volte vieni ignorato.

(5) Sii pronto per il rifiuto perché è più probabile che non venga rifiutato (anche se i tuoi risultati sono importanti, in realtà soprattutto se sono importanti).

Osservazioni:

(a) Mi piace pensare che la qualità delle recensioni sia associata alla chiarezza del documento, ma in passato ho ricevuto alcune recensioni discutibili che sembrano parole sparse che possono o non possono essere correlate al mio lavoro. Non serbare rancore e riprova felicemente.

(b) Se qualcosa è importante per te e sei finanziariamente capace, essere rifiutato una o due volte va bene finché continui a cercare di migliorare la tua presentazione e continua a comunicare con gli altri.

(c) Non vuoi uscire dalla faccia della terra e vivere come un eremita. Questo non aiuterà nessuno e soprattutto non te.

(d) Infine, sii aperto. Le persone commettono errori e talvolta la cosa che ha causato l'errore è significativa.

Laurent Duval
2016-09-24 17:38:08 UTC
view on stackexchange narkive permalink

Alcuni documenti molto importanti sono stati rifiutati per primi, altri non sono stati nemmeno pubblicati.

Se sei sufficientemente sicuro del tuo risultato e vuoi fissare una data per la tua scoperta, e non hai paura che a volte ci vuole tempo per dimostrare e convincere le persone della correttezza, metti il ​​tuo articolo online su qualche archivio aperto, mentre lavori a una versione che si spera pubblicata.

Alcuni lettori potrebbero scoprire un difetto e forse aiutarti a pubblicare.

Tom Au
2016-09-12 09:26:34 UTC
view on stackexchange narkive permalink

Prima di tutto, "risolvere famosi problemi aperti" non avviene nel vuoto. Deve esserci una buona ragione per cui te è venuta in mente la soluzione dopo aver eluso molti altri.

Una possibile ragione è che sei un esperto in qualche nuova tecnica o metodo di analisi. Allora il trucco sta nell'affermarsi come esperto in questo "nuovo" campo. Dopo averlo fatto, è più facile affermare che la tua padronanza di quest'area ti ha permesso di risolvere il problema "irrisolto" (a condizione che tu possa dimostrare la rilevanza del tuo campo). Ad esempio, se tu fossi un pioniere nella fisica subatomica che ha scoperto che la fisica newtoniana non ha funzionato nell'area subatomica, l'accettazione della tua "prova" dipenderà dall'accettazione da parte delle persone di te come esperto subatomico.

L'altra cosa è che se hai davvero scoperto una nuova soluzione a un problema, l'implicazione è che molto di ciò che è attualmente scritto sul campo in relazione a questo problema è sbagliato, o almeno deve essere ripensato. Il modo per metterti alla prova è iniziare a identificare a un livello basso e salire a livelli progressivamente più alti, le applicazioni che ora sono "annullate" dalla tua scoperta. Se puoi dimostrare che un intero "flusso" di idee deve essere ripensato e quindi presentare la tua scoperta come una soluzione "comune", le persone ti prenderanno molto più seriamente. Un esempio è stato quando le persone hanno capito che si poteva creare un nuovo sistema di geometria "non euclidea" semplicemente cambiando alcuni presupposti.
Ooker
2019-03-23 16:56:23 UTC
view on stackexchange narkive permalink

Preparati per la tua impavidità. Dovrai usarlo frequentemente.


Diciamo che il nostro obiettivo è quello di raggiungere l'elenco di Kaveh. Supponendo che abbia una certa esperienza nella ricerca accademica, ciascuna di queste affermazioni è equivalente:

  • La sua priorità non è realmente accettare il suo lavoro, ma in realtà non essere frainteso (l'effettiva premessa del domanda riguarda il feedback produttivo)
  • Per lui lei sta solo affermando l'ovvio (la parte superiore dell'elenco) e dicendogli di fare le cose che sta già facendo (la parte inferiore)
  • Ha bisogno di lui per completare la lista, ma non può farlo se lei lo accusa di aver fatto ciò che non ha fatto
  • Il problema di convincerla che non è un pazzo si riduce a i problemi emotivi e di incomprensione

Penso che in tutte le discussioni sugli eccentrici il presupposto schiacciante sia che sia arroganza. Questo è, sfortunatamente, unilaterale, perché le emozioni reali dovrebbero essere senso di colpa e frustrazione . È colpevole per aver preoccupato di essere arroganza e frustrato per il suo errore di valutazione che lui è arroganza. Con tutto il mio rispetto, penso che alla maggior parte degli esperti manchi l'esperienza necessaria per fornire un consiglio efficace.

Poiché questa risposta non cerca di rispondere ai problemi che la maggior parte degli esperti affermati desidera affrontare, è fondamentalmente una risposta frame challenge.

Il resto della risposta è solo un'elaborazione su come affrontare le reazioni emotive e le incomprensioni in ogni caso specifico. Ecco la tabella dei contenuti per questa risposta:

  • Gestire le tue emozioni
  • Gestire le loro reazioni
  • Altri problemi
  • Per coloro che vogliono aiutare
  • Link

Puoi anche leggere la risposta a una domanda generalizzata ( quali sfide si possono affrontare durante il progetto, non solo come convincere gli esperti). Presumo che la tua piattaforma per ottenere feedback sul tuo lavoro sia Reddit, ma non è un grosso problema.

Gestire le tue emozioni

Innanzitutto, ci sono due cose con cui cominciare:

  • Epistemologicamente, sembra che per risolvere un problema difficile, è necessario essere consapevoli della soluzione a priori
    L'epistemologia è un campo in filosofia studiando la natura della conoscenza e come la acquisiamo. Non ho letto molto su questo argomento.
  • Psicologicamente, quando sei consapevole di te stesso, queste emozioni autocoscienti evocheranno: orgoglio, vergogna, senso di colpa , e imbarazzo

Perché fare ricerca è dimostrare che hai torto prima di dimostrare di avere ragione, una cosa naturale quando pensi di sapere qualcosa è avere scetticismo su te stesso. Quindi, insieme al problema specifico con cui stai lavorando, farai queste domande consapevoli:

  • Sto affermando una cosa che non ho ancora dimostrato?
  • Affermo che so meglio degli esperti, quando mi manca l'istruzione formale?

Poiché le risposte a queste domande dovrebbero essere sì, evocheranno emozioni di autocoscienza, allontanandoti dal l'unica cosa su cui dovresti concentrarti: il problema su cui stai lavorando. Entrerai in un ciclo di feedback interrogandoti: la risposta a quelle domande è il motivo per cui ti metti in discussione in primo luogo. La vera risposta sta nel lavoro che stai ricercando, non nel sapere se sei sano di mente o no.

Per affrontare la fantasia (una sorta di orgoglio), prova a immaginare come sarebbe effettivamente la vita di una persona famosa. Per ogni tipo di successo che sogni, ci sono persone che l'hanno già ottenuto. Quindi dì che sogni il premio Nobel, poniti questa domanda: come viveva Einstein con la sua fama e i suoi soldi? Quando ti rendi conto che in realtà tutte le persone famose sono infastidite per essere famose, allora la tua fantasia è tagliata. Riuscendo a metterti nei loro panni, puoi staccarti dalle tue emozioni e tornare alla realtà. A quel punto, se ti chiedi ancora cosa farai quando diventerai famoso, l'unica cosa che desideri è un posto buio e tranquillo dove dormire senza sogni.

Tieni presente che potresti hanno complesso messia. Potresti non avere un'illusione grandiosa, ma il pensiero che (1) solo tu puoi fornire una soluzione che nessuno sembra vedere, e (2) in altri per fornire quella soluzione, devi superare lo scetticismo indipendentemente da quanto sia duro, può sviluppare questo complesso. Le persone potrebbero anche accusarti di aver eseguito gaslighting. Tutto ciò renderà ancora più forte il senso di colpa nei tuoi confronti.

Per distinguerli con veri problemi psicologici, la cui radice è l'insicurezza, li chiamerò "complesso di messia intellettuale" e "illuminazione a gas intellettuale" .

Perché la paura di essere deluso continua solo quando in realtà vedi che hai ancora fantasia, quindi quando l'hai tagliata con successo, la paura stessa scomparirà. Non proverai più vergogna, senso di colpa o imbarazzo per il tuo lavoro.

In generale, le emozioni sono intense solo quando la tua ricerca è ancora nella fase vaga. Più conoscenza acquisisci, meno frequentemente si presentano.

Gestire le loro reazioni

Nella mia esperienza, ci sono questi tipi di risposta improduttiva: etichettatura e sarcasmo.

Etichettatura

Quando ti dicono che sei eccentrico, dai loro questo indice pazzesco, elenca tutti i punti che potrebbero applicarti e spiega come si sbagliano. Se ti accusano di essere arrogante, digli che entrare con sicurezza in una tempesta con un sorriso è diverso dal cercare attenzione. Se pensano che tu sia pazzo (camminare in una tempesta con un sorriso è ovviamente pazzo), sarebbe molto più facile se tu potessi avere la conversazione faccia a faccia. Solo vedendo come i tuoi occhi sono determinati ma la tua mente non è affatto chiusa, che possono presumere che tu non lo sia. Non essere imbarazzante per aver detto loro come ti sei preparato per il progetto.

Parliamo di più sulla follia. Penso che sia meglio "aiutarli" a giungere alla conclusione che sei pazzo, perché non è più un'etichetta sul tuo comportamento, ma un'etichetta sulla tua razionalità. Quando raggiungerai quella fase, avranno una forte motivazione per continuare la conversazione e le tue prove saranno ascoltate attentamente. Se riesci a creare una dissonanza cognitiva nella loro mente, il loro pattern-matching rimarrà silenzioso e non saranno più bloccati nella loro prospettiva.

Rispondi alla loro etichettatura (stravagante, arrogante, folle, stupido , alto, pregiudizio, spam, insalata di parole, nemmeno sbagliato, woowoo, pseudoscienza, perdita di tempo, ecc.) fornendo la definizione della parola e mostrando perché non si applica a te. Avere una nota che elenca tutte le risposte preparate per ciascuna etichetta, in modo da non dover fare affidamento sulla tua scarsa memoria. Per affrontare lo scetticismo, devi immediatamente formare una combinazione perfetta di parole e il tuo cervello ti abbatterà. (Maggiori dettagli più avanti.)

Sarcasmo

Se fanno semplicemente delle battute, potrebbe essere vero che in realtà è divertente. In tal caso, forse è meglio continuare la battuta . Vedere come ti piacciono davvero le loro battute farà capire loro che non ti riguardano affatto. Ad esempio, se sei una persona grassa, scherzare sulla tua grassezza farà capire loro che sei consapevole del tuo difetto, ed è solo che non hai ancora il tempo per risolverlo. Se sei un cieco, fare una battuta sulla tua disabilità renderà i non ciechi sorprendentemente sorpresi. Ti percepiranno come invincibile ora.

Se fa davvero male e non riesci a pensare a una cosa intelligente da dire, forse è meglio ricordare loro che si trovano in un posto in cui deridere non è accettabile . Come uno spettacolo smack down, ti vedono letteralmente come il ragazzo arrogante e orgoglioso che ha bisogno di essere istruito. Annota i collegamenti a spettacoli comici, dove l'obiettivo è prendere in giro gli altri, come gli spettacoli della WWE, i video sulla teoria della cospirazione di higaniga o i subreddit beffardi, e mostra loro quanto siano esilaranti le loro azioni. Ad esempio, puoi dire:

Wow, r / Buddhism diventa r ​​/ WWE ora?

Oppure puoi invitare un'autorità a occuparsi di questo:

Non penso che questo comportamento sia appropriato. Segnalerò le mod / credo che anche altri non lo trovino appropriato.

In breve, sii intelligente quando si tratta di ad hominem. Il problema è che non puoi essere intelligente quando la tua mente è annebbiata dalla rabbia. Quindi devi prepararti per i momenti intelligenti. Quando riuscirai a farlo, la loro prossima risposta sarà probabilmente di nuovo produttiva.

Ma tutto sommato, non interagire con loro perché ti senti frainteso, ma a causa della conoscenza che hanno. Prova a convertire le domande sulla tua identità / te stesso (ad es. "Sono stravagante?") In domande sulla definizione (ad es. "Cos'è stravagante?"). Mostra come possiedi un corpo rigido di conoscenza di ciò che stanno cercando di trasmettere, con frasi come "psicologicamente parlando", "ciò di cui stai parlando è chiamato ______ in filosofia", "nel campo della logica informale", ecc. Sono campi accademici interessanti e rilevanti di cui non sono a conoscenza. Se pensi che abbiano la conoscenza di ciò di cui hai bisogno, concentrati solo su quello e ignora qualsiasi critica irragionevole nei tuoi confronti.

Quando hai risolto tutti i problemi emotivi, il 90% delle conversazioni ora sono produttive. Ma ci saranno ancora altri problemi.

Altri problemi

Problema commemorativo

Potresti aver perso così tanto tempo a leggere di crackpot, teorie del complotto, delusione, ecc. Potresti aver sprecato così tanto tempo nelle conversazioni che all'inizio le persone sono fortemente in disaccordo con te, ma dopo alcuni colloqui si scopre che non sono davvero in disaccordo con ciò che intendi realmente, e tutto questo è solo un malinteso. Prendi nota di tutte le prove che ti fanno sentire sulla strada giusta. Non lasciare che la tua ricerca sulla tua sanità mentale vada a buon fine.

Il problema di non essere in grado di spiegare te stesso è dovuto alla conoscenza tacita e alla punta della lingua fenomeno. Quando hai una prova molto forte che non sei un pazzo, dopo una settimana o giù di lì ciò che rimane è solo la sensazione che non lo sei. In mancanza di prove, verrai nuovamente trascinato nel giro. Devi creare immediatamente una perfetta combinazione di parole per uscirne.

Incomprensione

Anche quando accade un miracolo (passano il loro tempo e le loro energie per analizzare quello che dici), ci sarà un fenomeno molto strano che non puoi capire: senti sempre che sono implicitamente d'accordo con quello che dici, ma tu due non possono accontentarsi di un consenso, e quindi andare semplicemente in modo circolare.

Penso che tutti i conflitti nel mondo si riducano al problema della priorità. Di solito la situazione è così: la persona 1 può vedere che il problema A è più importante del problema B e la persona 2 vede che B è più importante di A. Il problema è che, la maggior parte delle volte sia A che B devono essere risolti insieme, altrimenti nessuno di loro può essere raggiunto. Ma poiché entrambi insistono sul fatto che il loro punto è più importante, entrambi si mancheranno e si supereranno a vicenda. Entrambi sentiranno che la conversazione è improduttiva e prima o poi la abbandoneranno.

Quando questo accade, è solo un cieco che guida un cieco, o peggio, un cieco combattimento. Investigare la natura di questo fenomeno e come affrontarlo è il mio interesse di ricerca. Il mio consiglio per affrontare il malinteso è di usare la negazione, non la spiegazione.

Egocentrismo

L'egocentrismo non riguarda l'egoismo o avere un ego enorme, ma non essere in grado di differenziare la tua mente da quella degli altri. Quando trovi qualcosa di interessante, assegnerai automaticamente che anche gli altri lo troveranno interessante e sarai confuso quando in realtà non gliene importa. Comunque ti ricordi di questo, questa tendenza si attiverà comunque.

Non dare per scontato che presumeranno che il loro feedback sia sbagliato. Potresti essere di mentalità aperta, e lo sono anche loro, ma in pratica non puoi farlo entrambi. Se presumi che siano curiosi di sapere perché hanno torto (una sorta di fiducia), tenderai a dare una spiegazione. Ma in realtà vedono solo che sei sulla difensiva. Ti accusano per cose che non hai mai fatto, quindi interrompono passivamente la conversazione o ti bloccano attivamente da ulteriori spiegazioni. Lo chiamo " trattamento del silenzio intellettuale".

In cerca di feedback

Quando viene fuori un'idea e ti senti benissimo, il tuo istinto te lo dirà ancora ci sono molti più campi che devi leggere attentamente. Sebbene non ti dispiaccia dedicare più impegno alla ricerca, vuoi solo chiedere un feedback perché sarebbe molto più efficiente. Il tuo egocentrismo presume che le persone otterranno ciò che ottieni anche tu. Se vuoi finire il progetto il prima possibile per passare ad altre cose importanti, la tua voglia di condividerlo sarà più alta. Ma STOP! Pubblicarlo ora riceverà solo critiche aspre e irragionevoli. Ascolta il tuo istinto e leggi prima tutti i campi di cui hai bisogno. Le buone domande arrivano solo quando la tua mente è nella fase dell'ignoranza, che è il risultato della comprensione del campo così com'è.

Detto questo, a un certo punto vedrai che non ha senso fare domande e vuoi solo che leggano il tuo lavoro così com'è. Se hai i requisiti necessari (revisione letteraria, citazione in linea, metodologia, ecc.), Ora puoi inviare la tua candidatura a una rivista accademica e non devi più aver paura di essere frainteso come un pazzo.

Ricezione di feedback

Nel caso in cui il pubblico principale del tuo lavoro sia il pubblico popolare, rendendo il tuo stile di scrittura necessariamente non accademico, la situazione sarà complicata, perché il tuo pubblico più importante è quello accademico. (Sì, a volte la cosa importante non è quella a cui dovresti dare la priorità - vedi il metodo Eisenhower.) Poiché il tuo lavoro deve servire due diversi tipi di pubblico, che hanno background, conoscenze e aspettative diverse, proverai sentimenti contrastanti quando riceverai il loro feedback:

  • Il pubblico popolare non può fornire un feedback utile, ma il suo entusiasmo indica che hai toccato un grosso problema che sta cercando
  • Lettori più esperti o persino accademici di lontano i campi possono fornire conoscenze utili e possono svolgere il ruolo di gatekeeper iniziali. Ma una volta che ti salutano, sai che non possono più aiutarti
  • Gli accademici dei settori pertinenti lo sentiranno vago o scialbo, perché si aspettano che il tuo lavoro sia veramente presentato in forma accademica. Ma se dici esplicitamente che questa è solo la fase per catturare ciò che hai in mente, il loro atteggiamento si ribalterà di 180 gradi

Parliamo di più dell'ultimo punto. Nella tua mente, non avere una revisione letteraria, una metodologia o un set di dati non è importante, perché accetti già di non averne uno. Pertanto, devi dire esplicitamente che l'articolo è solo una ricerca superficiale per tracciare una tabella di marcia per il tuo studio ed elencare tutte le tue carenze nel miglior modo possibile. Senza questa parte, le loro aspettative bloccheranno le loro menti e qualsiasi tua spiegazione d'ora in poi sarà percepita come difensiva.

Per coloro che vogliono aiutare

Ecco il mio consiglio per coloro che vogliono aiutare. Si spera che possa ridurre lo sforzo sprecato e portarti la massima felicità:

  • Presumi sempre che la persona con cui stai parlando abbia qualcosa di interessante che puoi imparare
  • Sii consapevole del fatto che anche se potrebbero non sapere di cosa stanno parlando, potresti non sapere nemmeno cosa vogliono veramente trasmettere
  • Conferma le loro osservazioni corrette prima di affrontare le tue preoccupazioni
  • Usa le domande socratiche

Non sentirti minacciato quando mostrano segni di irritabilità. Il fatto che accettino di essere etichettati come pazzi indica che hanno qualcosa di più importante da fare. Come te, sono creature razionali e hanno già eseguito un'analisi costi-benefici prima di iniziare il progetto. Quando vuoi dare loro dei consigli, potresti volerlo inquadrare in questo modo:

  • Non penso che ci sia un'altra opzione per fare X / Il modo migliore per farlo è Y. Ma visto che sembra che tu lo sappia anche tu, puoi spiegare perché non pensi che ti si addica?
  • Non credo che tu abbia capito il concetto Z così com'è. Secondo me, Z riguarda a, b o c, e forse c è più vicino a ciò che intendi. È corretto?

Potresti voler conoscere le metafore concettuali se vuoi sapere perché a volte le idee serie sono esilaranti. Consiglio il libro Metaphors we live by di Lakoff e Johnson.

Link

Kletische ha alcuni buoni articoli su questo argomento:

Puoi anche leggere la mia ricerca: Una teoria della prospettiva. Discute di varie cose, due delle quali sono il tradimento intellettuale e lo sguardo freddo, che sono rilevanti per questa risposta.

Che risposta assurdamente lunga.Eppure sembra contenere a malapena qualcosa di utile all'OP, essendo per lo più privo di significato.
le altre risposte presumono che il ricercatore possa mantenere una mente fredda a fondo e gli esperti non danno una singola derisione alle prime idee, ma danno solo un feedback o rimangono in silenzio.Questo non è il caso.Se possibile, puoi spiegare perché le prime due sezioni sono prive di significato?
pensaci, forse solo le prime due sezioni direttamente correlate alla domanda (come convincere).Gli altri punti riguardano più uno generalizzato (quali sfide si possono affrontare).Allora li tolgo
Questo non risponde alla domanda.Questa risposta descrive come l'OP dovrebbe affrontare la propria salute mentale / emotiva (il che è importante, ma il consiglio dichiarato non è valido) e fornisce lezioni alla comunità di ricerca su come dovrebbero trattare gli estranei (che è inclinato ai mulini a vento), nessuno dei quali indirizzoLa domanda di OP su come ** possono ** convincere al meglio ** altre persone ** che la loro soluzione è legittima.
_Quando ti dicono che sei eccentrico, dai loro questo indice pazzesco, elenca tutti i punti che potrebbero applicarti a te e spiega come si sbagliano._ - Questo potrebbe essere il modo meno efficace per convincere qualcuno che non sei unmanovella che io abbia mai letto.
@JeffE Si scopre che questa risposta è una [sfida frame] (https://interpersonal.meta.stackexchange.com/q/2511/86), in cui è necessario un consiglio apparentemente fuori tema.Aggiungo la spiegazione su questo in alto, potete verificarlo?
Scusa no.Troverei la sfida del telaio appropriata se OP avesse chiesto "Sono stato etichettato come una pedivella; come faccio a convincere le persone che non lo sono?"o "Come gestisco gli attacchi ad hominem alla mia ricerca?", ma non è quello che hanno chiesto.(E il consiglio che offri è ancora spento.)
@JeffE Qual è la differenza tra la tua prima domanda di esempio e la domanda del titolare?E in ogni caso, che tipo di domanda pensi che questa risposta sia appropriata e buona?
La differenza è che OP non dice di essere stato definito un pazzo o disprezzato in alcun modo.Stanno cercando di _prevenire_ tali attacchi in primo luogo, non di _recuperarli_ da loro.
@JeffE Cercare di prevenirlo porterà alla [profezia che si autoavvera] (https://en.wikipedia.org/wiki/Self-fulfilling_prophecy).Quando un principiante afferma di avere una grande idea, a causa della sua mancanza di gergo, deve usare l'analogia per esprimerla.Questo renderà l'idea così assurda e divertente per gli esperti, quindi prima o poi si farà ridere.Per questo motivo, è meglio aspettarsi un attacco ad hominem ed equipaggiare le abilità per affrontarlo, piuttosto che cercare di prevenirlo a tutti i costi ed essere deluso quando arriva.
Hai appena risposto alla domanda: ** Segui la lingua e le abitudini del pubblico che vuoi convincere. ** Se non conosci la lingua, non usare solo "analogie" - _impara_ la lingua.Leggere.Studia.Ascolta.Essere pazientare.
@JeffE È complicato, ma posso assicurarti che essere paziente è semplicemente impossibile.È come la frustrazione che non vuoi essere infedeltà, ma hai ancora l'erezione sulle ragazze sexy.Penso che un principiante (1) veda effettivamente qualcosa che gli esperti del settore non hanno visto e (2) non abbia ancora letto libri di testo, l'unico modo in cui ciò è possibile è che (A) è una ricerca interdisciplinare(B) si basa su un'analogia.Sia A che B fanno sì che sia il ricercatore che l'esperto non sappiano nemmeno quali campi dovrebbe guardare, e quindi entrambi presumono che il suo campo sia quello corretto.
Per non presumere che lui stia parlando del suo campo, e quindi non si senta insultato nel suo campo, deve prima conoscere il campo che dovrebbe leggere.
_ Posso assicurarti che essere paziente è semplicemente impossibile.È come la frustrazione che non vuoi essere un'infedeltà, ma hai ancora l'erezione sulle ragazze sexy._ - E posso assicurarti che la pazienza ** non è solo possibile, ma comune.È come essere eccitato da ragazze sexy e poi non aggredirle.
@JeffE Puoi rimanere calmo solo se hai la certezza che il tuo amore per il tuo partner sia vero, o addirittura insostituibile.Se, per qualsiasi motivo, c'è una crisi che ti fa dubitare del tuo amore per lei, nonostante il fatto che tu lo faccia effettivamente, allora qualsiasi eccitazione che otterrai aggiungerà di più alla tua ansia esistente.La calma / la pazienza non possono essere raggiunte quando ci si sente in colpa.Per "impossibile" intendo il [carico cognitivo] (https://en.wikipedia.org/wiki/Cognitive_load).Se consideriamo la lettura dei libri di testo come nostro obiettivo, voglio solo esaminare quali fattori possono impedirlo.


Questa domanda e risposta è stata tradotta automaticamente dalla lingua inglese. Il contenuto originale è disponibile su stackexchange, che ringraziamo per la licenza cc by-sa 3.0 con cui è distribuito.
Loading...